Você está na página 1de 77

PART I: ACUTE CARE SURGERY

Chapter 1: Hernias → SBO


Differential for groin mass
-most often groin mass will be a lymph node: autoimmune vs malignancy vs physiologic/reactive
-reactive: usually sub-centimeter, mobile, tender, firm; follicular hyperplasia in response to minor infections, cuts, scrapes, etc
in local region or lower extremity
-think syphillis, chancroid, LGV if large, tender, and just in inguinal region
-malignant: large non-tender, limited in inguinal region
-systemic process: large non-tender, associated with diffuse LAD
-abscess: from boils, furuncles, hidradenitis suppurativa
-femoral aneurysms (rare): trauma or prior interventions, feel for pulsatility
-scrotal masses in men

H&P
-mass protrudes with straining (Valsalva), reduces with supine positioning
-prior incisions, heredity, multiple pregnancies, obesity, liver disease+ascites
-chronic straining, chronic cough (COPD, lung cancer), constipation (colon cancer), urinary strain (BPH, prostate cancer), heavy lifting

Pathophys
-definition: protrusion of tissue or organ through a defect, most commonly in the abdominal wall
-three components of a hernia: the abdominal wall defect, protruding hernia sac, and the contents of the sac
-Kinds:
-reducible: can be pushed back through defect into peritoneal cavity
-incarcerated: contents stuck in hernia sac
-strangulated (surgical emergency): type of incarcerated hernia where blood flow is compromised, <1% of all inguinal hernias
-close-loop bowel obstruction: both ends are blocked, fluid/gas is produced but cannot exit → distention, further
compromise of blood flow
-signs: ischemic bowel causes fever, tachy, elevated WBC (SIRS), erythema
-Types:
-direct: acquired, weakness of transversalis fascia through Hesselbach’s triangle (rectus sheath, inferior epigastric vessels,
inguinal ligament), only goes through the superficial ring
-indirect: congenital, persistent processus vaginalis (supposed to descend into scrotum, testicles go down with spermatic cord),
goes through superficial and deep inguinal rings
-if patent and fills with fluid → hydrocele
-if patent and bowel goes down → indirect hernia
-femoral (below inguinal ligament): females, more likely to strangulate given narrow, rigid canal
-umbilical: pediatric, congenital hypothyroidism, usually asymptomatic, close spontaneously by age 2, in adults from increased
intra-abdominal pressure
-ventral/incisional: from previous surgery
-Richter’s hernia: only part of the bowel wall circumference is trapped
-sliding hernia: when a retroperitoneal organ (bladder or colon) moves with hernia and becomes the posterior wall of the hernia
sac
-Spigelian hernia: lies between two layers of abdominal wall

Workup and Management


-clinical diagnosis
-indirect hernias are lateral to the inferior epigastrics, direct are medial

Page 1 of 77
-imaging
-US with straining
-CT scan
-surgery: mesh for indirect, direct, and femoral hernias
-indirect: open sac anteriorly, reduce hernia, do a high ligation at internal ring
-direct: don’t open sac
-strangulated is a surgical emergency but incarcerated should try to reduce, if can’t reduce then surgical intervention should be scheduled
(semi-elective)

Complications/Trouble

-not recognizing Richter’s → no signs/symptoms of SBO but can still be strangulated


-not recognizing sliding hernia → bowel/colon can be injured/transected
-not separating vas deferens/testicular vesicles in spermatic cord from the hernia sac

Controversial
-asymptomatic hernia repair → watchful waiting, femoral hernias are recommended since easily strangulated
-in a premie wait until out of NICU
-incisional/ventral hernia repair → asymptomatic?, don’t want re-operation
-lap repairs have slightly higher recurrence rate than open repair?

Chapter 2: adhesions → SBO


Differential for abdominal pain + nausea/vomiting

H&P
-abdominal scar  previous surgery with adhesions
-hernias most common cause of SBO
-Howship-Romberg sign: obturator hernia with compression of obturator nerve (pain on medial aspect of thigh, with abduction,
extension, internal rotation of hip), seen in elderly, multiparous females, or those with significant weight loss
-severe abdominal pain with localized tenderness in SBO  strangulated hernia (fever, tachy, ↑WBC, localized tenderness)

Pathophys
-closed loop obstruction: segment of intestine obstructed proximally and distally, gas/fluid accumulate, risk of strangulation with
ischemia/perforation
-SBO  dilation of bowel proximal to obstruction, increased motility in attempt to overcome causes colicky pain, high-pitched/tinkly
sound at the beginning then later no bowel sounds
-in USA adhesions (appy, colorectal, gyn surgeries)
-dehydration in SBO: caused by emesis, transudative loss into peritoneal cavity, also third spacing (fluid into bowel wall with dilation
and increasing pressures in the segment proximal to obstruction)

Page 2 of 77
Workup
-lab: CBC, BMP, lactate
-abdominal series:
-upright CXR – free air
-upright KUB – air/fluid levels
-supine KUB – dilated bowels
-CTAP with IV and PO contrast
-complete vs partial SBO: same symtoms but partial SBO will be slower developing symptoms and can still pass some gas
-partial SBO plain films will show residual colonic gas
-partial SBO can be non-operative management
-LBO vs SBO: LBO symptoms are more gradual symptoms
-postop ileus vs SBO: usually small intestine regains motility in 24 hours, stomach 48hours, colon 3-5days
-ileus: dilated loops of bowel but no air/fluid levels

Management
-aggressive fluid/lytes
-NGT
-foley
-partial SBO: fluids+NGT, if progress  OR
-complete SBO: can do close observation if no signs of ischemia/peritonitis
-viable vs dead bowel: pink vs pale/blue color, peristalsis, Doppler for artery, IV fluorescein dye + Wood’s lamp
-postop SBO: avoid re-operation for 10-14days postop unless there is peritonitis/bowel compromise, ideally conservative management
until 4-6 weeks postop

PART II: BREAST

Chapter 3: new breast mass


Differential for benign breast masses

Differential for malignant breast masses

Page 3 of 77
H&P
-hard, immobile, irregular borders, cervical/supraclavicular/axillary LAD
-risk factors for breast cancer: female, age, fam hx of breast cancer, bilateral breast cancer, ovarian cancer, BRCA1/2, DES exposure,
early menarche, nulliparity or childbirth after age 30, late menopause, obesity, hormone therapy
-nipple discharge:
-benign: clear, bilateral, multiductal
-physiologic: up to 2 years after giving birth, puberty/menopause, nipple stimulation
-pathologic: prolactinoma, hypothyroidism, Cushing’s disease, meds (antipsychotics, cimetidine)
-unilateral bloody  papilloma
-cancer: bloody, spontaneous, unilateral, uniductal, associated breast mass, >40yo

Pathophys
-fibrocystic change is benign: high risk with ductal hyperplasia/sclerosing adenosis and atypical hyperplasia
-peu d’ orange: compromised lymph drainage, edema of interfollicular dermis, mostly in inflammatory carcinoma  need punch biopsy
of dermis
-nipple retraction  tumor infiltrates suspensory ligaments of Cooper

Workup
-triple test for new breast mass: physical exam, imaging, biopsy
-if <30yo, get US: can differentiate between cyst or solid mass, can use for biopsy
-simple cyst  observe
-enlarging/painful cyst  aspirate, send bloody fluid to cytology
-solid: biopsy
-if >30yo, get mammogram  if mass, US guided core needle biopsy
-metastatic workup: LFTs (PAB- PT, Alb, billi), bone (alk phos, serum Ca), CXR
-if think its advanced breast cancer can get CT chest, abd/pelvis, and bone scan
-AJCC staging

Page 4 of 77
-triple negative (ER, PR, HER2) most common subtype in BRCA 1 carriers, have high histological grade, occur at younger age, and
more common in AA females

Management
-Surgical options: BCT (lumpectomy, SLNB, radiation) vs simple mastectomy with SLNB
-BCT only for stages I and II, risk of recurrence
-cannot do BCT if multiple tumors in multiple quadrants, diffuse microcalcifications, previous chest wall radiation, positive
margins, early pregnancy (no radiation in pregnancy) but can do in third trimester and radiate after birth
-if stage III: mastectomy and ALND or radical mastectomy with ALND, can give neoadjuvant chemo

-get echo or MUGA scan for ejection fraction before trastuzumab


-oncotype DX looks at tumor molecular stuff

Complications
-dissection nerve injuries

-most morbid complication is lymphedema, can progress to lymphangiosarcoma

Trouble
-differentiating between inflammatory breast cancer and cellulitis
-missing a breast mass during pregnancy, can do chemo during 2nd trimester, radiation and hormonal therapy never given during
pregnancy
-ignoring breast mass in elderly male
-risk factors: hormonal therapy, obesity, alcohol/cirrhosis, Klinefelter, BRCA, fam hx, radiation therapy
Page 5 of 77
-gynecomastia is not a risk factor!

Chapter 4: abnormal screening mammogram


Differential for abnormal mammogram without a palpable mass

Screening
-age 40, annually
-clinical examination age 20-39 every 3 years

Pathophys
-larger calcifications are almost always benign, microcalcifications are more seen with breast cancer
-DCIS vs LCIS
-DCIS: several histological patterns, comedo-type is worst, can be multifocal, and associated with risk of invasive carcinoma
-LCIS: not a premalignant lesion, not thought to progress to invasive carcinoma, but risk of developing invasive
carcinoma(ductal or lobular) is 2x higher than those without LCIS

Workup
-mammogram BIRADS

-BIRADS 6 = known malignancy

Management
-DCIS  lumpectomy with negative margins
-LCIS  rule out adjacent cancers, monitor carefully after with serial mammograms and physical exams
-IDC  lumpectomy with SLNB and radiation vs mastectomy with SLNB

Page 6 of 77
PART III: CARDIOTHORACIC

Chapter 5: chest pain (ACS)  STEMI vs NSTEMI vs UA


Differential

H&P
-risk factors: age, male, HTN/HLD, DM, smoking, fam hx of CAD
-classic hx: chest pain, diaphoresis, anxiety, tachy, SOB, nausea/vomiting, JVD, crackles, new murmurs
-postmenopausal women at higher risk for MI, with atypical symptoms/presentation usually

Pathophys
-UA: non-occlusive thrombosis causing reduced myocardial perfusion, no necrosis  no elevation in cardiac enzymes
-NSTEMI: occlusive thrombosis eliminates perfusion to partial thickness of myocardial wall (subendocardial)  elevation in cardiac
enzymes
-STEMI: occlusive thrombosis eliminates perfusion to full thickness myocardium  elevation in cardiac enzymes, ST segment elevation
on EKG
-LAD is most commonly affected
-causes can include plaque rupture, decreased oxygen supply, increased oxygen demand
-prinzmental angina: unrelated to exertion in young person, coronary artery vasospasm for transient decreased perfusion, can see ST
segment elevation, give Ca-channel blockers

Workup
-EKG
-LAD  V2, V3, V4
-circumflex (lateral/posterior of LV/LA)  aVL, V5, V6
-posterior descending artery (inferior LV, posterior septum)  II, III, aVF
-cardiac enzymes
- troponin-I and CKMB q8hrs for first 24hours (troponin and CKMB renally cleared, will be high in CKD)
-CKMB for secondary MI
-CXR
-rule out pneumothorax, dissection, etc

Management
-STEMI: PCI within 90mins, tPA if >90mins or no PCI, CABG if PCI fails
-aspirin, clopidogrel, platelet IIb/IIIa antagonist, heparin
-beta blocker
-nitroglycerin
-morphine
-ACE inhibitor, ARB
Page 7 of 77
-atorvastatin
-oxygen
-NSTEMI:
-aspirin, platelet IIb/IIIa antagonist, heparin
-beta blocker
-nitroglycerin
-morphine
-ACE inhibitor
-oxygen
-PCI if recurrent angina, CHF, LVEF <40%, hemodynamic instability, V-tach, prior CABG
-elective CABG: give aspirin, beta blockers, statins, abx prophylaxis
->50% stenosis of LAD
->70% stenosis of other coronaries
-stenosis of LAD + 1 other artery
-stenosis of 3 coronaries
-if obstruction cannot be stented (at bifurcations)
-graft: internal mammary artery, greater saphenous vein, radial artery
-CABG is better than stenting

Complications

Trouble
-silent MI: most often in women, elderly, diabetic patients
-aortic dissection Stanford A (ascending aorta) can also cause coronary artery occlusion and an MI

Chapter 6: chest pain, murmur  aortic stenosis


Differential for chest pain

Page 8 of 77
H&P
-S4 heart sound means LV hypertrophy seen in HTN, HOCM, aortic stenosis
-aortic stenosis: crescendo-decrescendo systolic murmur, decrease with Valsalva, classic triad of angina, syncope, CHF
-pulmonic stenosis
-HOCM will decrease with stooping/leg raise, increase with Valsalva
-new murmur with IV drug use think endocarditis, petechiae, splinter hemorrhages in nail, osler nodes in skin (tender), janeway lesions
(nontender) in palms, soles, finger pads, roth spots

Pathophys
-aortic stenosis (also risk of sudden death if symptomatic)
-senile: after 6th decade of life, calcification of normal valve, most common cause
-congenital: middle aged, calcification and fibrosis of bicuspid aortic valve
-rheumatic valve: immigrants, either aortic or mitral valve

Workup
-BMP, cardiac enzymes, BNP, d-dimer
-CXR
-EKG
-echo
-diastolic, continuous murmurs always get an echo!

Management
-mild: treat HTN and other risk factors, monitor with echo every 3-5years
-valve replacement if symptomatic
-severe aortic stenosis caution with blood pressure medications (beta blockers, diuretics, vasodilators, etc)
Controversial Areas
-bioprosthetic (animal tissues, don’t last as long, can calcify) vs mechanical (longer lasting, but lifetime anticoag)
-ross procedure: using own pulmonary valve as aortic, pulmonary replaced from deceased individual, no anticoag required, usually for
children
-percutaneous balloon valvuloplasty if poor surgical candidates, temporary fix, bridge to surgery
-percutaneous aortic valve replacement if poor surgical candidate, not durable, risk of stroke

Chapter 7: chest and back pain  aortic dissection (Marfan’s)


Differential

H&P
-risk factors for aortic dissection: HTN, Marfan/EhlersDanos, age, atherosclerosis, pregnancy, cocaine, aortic injury, aortic coarctation
-uneven limb pulses depending on where dissection occurs, arch vs descending aorta
-increased pulse pressure  aortic insufficiency

Pathophys
-tear in intima, then media, and adventitia alongside original vessel, producing false lumen

Page 9 of 77
-false lumen can enlarge, re-enter original vessel, or tear
-Marfan’s (autosomal dominant) – defect in fibrillin  cystic medial necrosis/weakness
-aortic aneurysm is a dilation of all 3 layers of vessel (Marfan’s also at risk for aortic root aneurysm)
-stanford A dissection involves ascending aorta and arch, Stanford B is descending aorta (distal to left subclavian)
-stanford B most common but Marfan’s more with Stanford A
-complications of dissections
-can cause malperfusion syndrome (false lumen can interfere with blood flow in branching vessels), can be to renal artery,
spinal arteries (paraplegia), SMA (abdominal pain out of proportion to exam), limb ischemia
-type A dissection: can interfere with coronary artery flow or carotid arteries or dissect into pericardial sac
-type B dissection: can extend a long distance without causing malperfusion syndrome

Workup
-type and cross
-EKG
-cardiac enzymes
-CXR (widened mediastinum)
-chest CT if CXR inconclusive but high suspicion
-OR immediately if HDS unstable, do TEE and can start surgery

Management
-beta blockers to lower HTN; if significant aortic regurge or tamponade then no beta blockers
-can cause hypotension and cardiac arrest!
-if type A  beta blocker, OR
-mortality 50% in first 48hours if no surgery
-if type B  beta blocker, nitroprusside, Ca-channel blocker, morphine, ICU, monitor BP, OR if complications arise (malperfusion
syndrome, or limb/visceral ischemia)
-80% survive with appropriate treatment
-complication of repair of dissections of thoracic aorta  spinal cord ischemia from artery of Adamkiewicz

PART IV: ENDOCRINE

Chapter 8: adrenal incidentaloma


Differential

H&P
-secreting cortisol: truncal obesity, moon facies, buffalo hump, proximal muscle wasting, purple striae, easy bruising, osteoporosis, poor
wound healing, women can have amenorrhea and hirsutism
-exogenous/iatrogenic: immunosuppression
-adrenal: adenoma, carcinoma, bilateral hyperplasia
-pituitary (Cushing’s disease): ACTH hypersecretion
-lung: non-small cell lung cancer (ectopic ACTH), bronchial carcinoid

Page 10 of 77
-aldosterone (Conn’s syndrome)  HTN with hypokalemia
-pheochromocytoma  HTN, headache, flushing, palpitations
-if adrenal carcinoma most commonly present with cushing’s syndrome, virilization, feminization, can have abdominal mass, abdominal
pain

Pathophys
-zona glomerulosa is aldosterone, zona fasciculata is cortisol, zona reticularis is androgens (DHEA, DHEA-S, androstenedione), adrenal
medulla is catecholamine
-cortisol increases blood glucose (inhibits insulin mediated glucose uptake, stimulates glycogenolysis, stimulates gluconeogenesis,
proteolysis, lipolysis), also increases renal absorption of Na, raises peripheral vascular resistance causing HTN, also impairs cellular
immunity increasing risk of infection

Workup
-24hour urine cortisol, low-dose suppression test, can use high dose suppression test to distinguish between pituitary or ectopic ACTH
(pituitary is suppressed, ectopic is not)
-plasma aldo: renin ratio (normal 4-10), serum K
-24hour urine or plasma metanephrines/catecholamines
-all functional masses, resect
-if nonfunctional adrenal mass <4cm monitor with interval CTs, if 4-6cm surgery if low risk candidate, if >6cm or features of malignancy
(>6cm, irregular borders, necrosis, calcification, hemorrhage, invasion into nearby structures, low levels of intracellular lipid, high
vascularity) then resect
-if won’t resect, follow up CT imaging at 6, 12, 24 months and hormone blood levels every year for 4 years

Management
-if Cushing’s syndrome, pre-op need to give glucocorticoid replacement since ACTH has been suppressed
-if aldo mass, pre-op give spironolactone and potassium
-if pheo, pre-op give alpha blockade 10-14days prior to surgery, beta blockers only after good alpha blockade
-percutaneous biopsy not required for functional tumors based on biochemical analysis
-don’t do until ruled out pheo
-don’t do if suspicions for adrenocortical carcinoma since can seed the biopsy tract
-would only do if gauging metastatic disease from known extra-adrenal primary malignancy

Chapter 9: hypercalcemia  hyperparathyroidism


Differential for hypercalcemia

Page 11 of 77
-PTHrP (squamous cell cancers), ACTH (non-small cell)

H&P
-hypercalcemia: stones (kidney), bones, abdominal groans (constipation, pain, n/v), neuropsych moans (fatigue, depressed, difficulty
concentrating, impaired memory
-risk factors for primary hyperparathyroidism: radiation, fam hx, lithium therapy for bipolar
-MEN syndromes
-MEN 1: hyperparathyroidism, pituitary adenoma, PNETs
-MEN 2A: marfanoid habitus, hyperparathyroidism, medullary thyroid cancer, pheo
-MEN 2B: marfanoid habitus, oral neuromas, medullary thyroid cancer, pheo
-Chvostek’s sign: facial twitch when tapping on facial nerve anterior to ear  hypocalcemic tetany after parathyroidectomy
-Trousseau’s sign: blood pressure cuff, flexion of wrist and MCP and digit extension  hypocalcemic tetany
-T score <-2.5 is osteoporosis
-anatomy:
-superior glands from 4th pharyngeal pouch migrate with lateral anlage of thyroid
-inferior glands from 3rd pharyngeal pouch, migrate with thymus
-ectopic locations behind esophagus or prevertebral fascia (superior glands), in mediastinum or carotid sheath (inferior glands)

Pathophys

-primary hyperparathyroidism  adenoma or all 4 glands if gene mutation (MEN)


-secondary hyperparathyroidism  CKD or vit D deficiency which causes hypocalcemia so ↑PTH
-tertiary hyperparathyroidism = persistent excess secretion of PTH after renal transplant, this is rare

Workup
-labs: Ca, P, Cl, bicarb, Mg, serum crt, PTH, 24hour urine Ca
-need high PTH, high serum Ca, high urine Ca to diagnose primary hyperparathyroidism
-can also have a Cl:P ratio >33 in primary form (hyperchloremic metabolic acidosis)
-need to rule out hypocalciuric hypercalcemia (familial), will have low urine Ca
-order DEXA, can get bone xrays
-99m technetium sestamibi scanning and ultrasound to localize which gland

Management
-surgery is indicated if symptomatic (stones, bones, abd groans, or psych moans)
-if asymptomatic primary hyperparathyroidism, surgery if
-serum Ca >1.0 than ULN
-creatinine clearance <60mL/min
-DEXA <-2.5
-age <50
-poor follow-up
-secondary hyperparathyroidism, surgery if PTH >800, symptoms, progressive renal disease, fractures, calciphylaxis
Page 12 of 77
-nonoperative management: bone mineral density 1-2years, bisphosphonates, cinacalcet
-surgery
-if single adenoma, remove the 1 enlarged gland
-if all 4 hyperplasia then remove 3.5 glands or 4 glands and reimplant ½ gland into brachioradialis muscle or SCM
-post-op: watch for neck hematoma (bedside decompression then OR), voice changes (recurrent laryngeal nerve injury), perioral
numbness, tingling in fingers (hypocalcemia), give PO calcium supplementation if necessary
-post-op hypocalcemia: bone may absorb a lot of calcium right after surgery after having high PTH

Trouble Areas
-normal Ca does not rule out hyperparathyroidism, need to take into account albumin
-need to check Mg level if someone is not responding to Ca supplementation
-treating hypercalcemic crisis  aggressive IV NS, then loop diuretics (don’t give thiazides!), can use dialysis if renal failure, can add
bisphosphonates
Chapter 10: episodes of sweating and palpitations, and HTN  pheochromocytoma
Differential

-surgery for HTN: pheo, adrenal adenoma (cortisol or aldo), renal artery FMD, or aortic coarctation

H&P
-classic triad for pheo: headache, flushing, palpitations
-other symptoms are HTN refractory to meds, anxiety, impaired GI motility, MI, arrhythmias, stroke, even HF
-orthostatic hypotension can also be symptom from desensitized adrenergic receptors, normal baroreceptor reflex is gone
-usually diagnosed in 40-50yo, M=F
-rule of 10s of pheos = 10% bilateral, 10% extra-adrenal, 10% familial, 10% multifocal, 10% malignant
-make sure to take BP in both arms and legs to rule out coarctation
-syndromes with pheos
-MEN2A: medullary thyroid cancer, primary hyperparathyroidism (50% risk)
-MEN2B: medullary thyroid cancer, mucosal neuromas, marfanoid habitus (50% risk)
-neurofibromatosis type 1: neurofibromas, café au lait spots, Lisch nodules (1-5% risk)
-von Hipple Lindau: retinal angioma, CNS hemangioblastoma, renal clear cell cancer, PNET, pancreatic and renal cysts (10-
20% risk)
Pathophys

Page 13 of 77
-tumor from chromaffin cells in adrenal medulla or anywhere along sympathetic ganglia
-norepinephrine > epinephrine, dopamine is rare
-if dopamine is present will have watery diarrhea
-HTN crisis or catecholamine release can be caused by changes in blood flow, physical stimulation, tumor necrosis, certain drugs,
anesthetics, can also be triggered by intubation, foods with tyramine, trauma, surgical manipulation, stress, biopsy attempts
-can have elevated hematocrit from chronic alpha constriction, also can have high EPO as paraneoplastic
-can have hyperglycemia since catecholamines stimulate hepatic glucose production and inhibit insulin secretion
-malignant pheo means it is invading into adjacent organs or vessels or have metastasis
-malignancy not based on histopathology analysis
Workup
-24hour urine for catecholamines or metanephrines, need to be 2x ULN
-interfering medications (false elevations of catecholamines): beta blockers, decongestants, antidepressants (TCAs, MOA inhibitors,
venlafaxine), antipsychotics (clozapine)
-don’t biopsy a pheo!
-CT scan or MRI
-nuclear imaging with I-MIBG

Management
-alpha blockade (phenoxybenzamine) 2 weeks prior to surgery, then beta blockade, then resection
-if give beta blocker first, unopposed alpha will cause constriction  HTN crisis
-genetic testing/counselor recommended for all pheos
-measure plasma/urinary catecholamines 2 weeks postop, every 3 months for 1 st year, then annually after
-complications
-watch out for Addison’s if bilateral adrenalectomy, check ACTH
-HTN if residual tumor or metastatic
-hypotension if loss of catecholamine induced vasoconstriction, or residual alpha blockade
-hypoglycemia if rebound hyperinsulinemia
-arrhythmia if had HTN crisis
-prognosis if malignant pheo: no cure, management is medical palliation of symptoms

Chapter 11: neck mass that moves with swallowing  thyroid mass
Differential for thyroid mass

Page 14 of 77
H&P
-lump in anterior neck, can have SOB, neck tightness, voice changes, dysphagia
-risk factors for thyroid cancer: female, exposure to ionizing radiation (especially before 15yo), fam hx, MEN2A/2B, familial medullary
thyroid carcinoma (FMTC), RET/PTC rearrangements/mutations, BRAF mutations
-symptoms of hyperthyroidism: nervousness, weight loss, heat intolerance, thirst, palpitations, tremors, hyperactive reflexes, irregular
menses, thyroid storm (high fever, arrhythmia, GI upset)
-symptoms of hypothyroidism: fatigue, weight gain, lethargy, hair changes, puffy face, cold intolerance, constipation, difficulty with
memory/cognition, impaired fertility

Pathophys
-Tg is the glycoprotein within the follicles that store T4 and T3, can be used as a tumor marker after initial treatment of cancer, however
anti-Tg Ab can occur in cancer and interfere with Tg monitoring
-ectopic thyroid tissue can be anywhere along thyroglossal duct, in anterior mediastinum, or in lateral neck if it is metastasized to cervical
lymph nodes
-FNA cannot diagnose follicular thyroid carcinoma (need a histologic sample to see invasion of capsule) but can use to diagnose papillary
cancer
-psammoma bodies seen in papillary thyroid cancer (also serous cystadenocarcinoma of ovary, meningioma, mesothelioma)

Workup
-TSH
-ultrasound
-CT or MRI if evidence of locally advanced thyroid cancer
-thyroid scintigraphy with radioactive I123 (rarely used)  hot nodules are benign, cold are malignant
-FNA if >1cm, or with features on US suggestive of malignancy such as microcalcifications, or history of growth
-also FNA suspicious cervical LAD if present
-no FNA if <1cm, purely cystic, benign features on US

Management
-inadequate/nondiagnostic FNA  repeat biopsy

Page 15 of 77
-benign FNA  annual f/u with US, repeat FNA or excise if nodules enlarge or develop suspicious features
-atypia of undetermined significance or follicular lesion of undetermined significance (AUS/FLUS) repeat FNA
-follicular neoplasm  remove on lobe of thyroid (need to distinguish adenoma vs adenoCA)
-if lobectomy shows cancer  total thyroidectomy so can avoid missed foci of cancer and also now can use radioactive iodine
scan to detect/treat recurrence of metastasis
-spreads hematogenously, no need for lymph node dissection
-if papillary thyroid carcinoma <1cm  can just do lobectomy (low recurrence rate)
-if >1cm papillary carcinoma total thyroidectomy
-lymph node dissection is controversial
-medullary carcinoma  total thyroidectomy
-central lymph node dissection if >1cm carcinoma
-anaplastic thyroid cancer  no standardized therapy, surgical resection, chemo, radiation, usually palliative treatment
-post-op: radioactive iodine ablation, levothyroxine to supplement hormone and to suppress TSH
-baseline Tg and anti-Tg Ab 2x/year, 1x/year if stable, annual neck US
-if suspect recurrence, get neck US

Complications
-during surgery, injury: recurrent laryngeal nerve, superior laryngeal nerve, parathyroid glands
-recurrent laryngeal nerve: muscles in larynx, sensory below vocal cords, parasympathetic to mucosa so will have paralyzed
vocal cord resulting in hoarseness and sometimes aspiration; if bilateral can cause airway obstruction and may need
tracheostomy
-external branch of superior laryngeal nerve – cannot speaking/singing in high pitch
-parathyroid glands then give calcium
-neck hematoma  stridor in postop, tense neck wound  cut sutures and open wound at bedside for decompression

Trouble Areas
-Fam hx, especially if medullary
-also need to get 24hour urine catecholamines/metanephrines to exclude pheo before surgery
-don’t miss postop hypocalcemia
-thyroid tissue in lateral neck means metastatic papillary thyroid carcinoma

PART V: HEAD AND NECK

Chapter 12: progressively hoarse voice


Differential

H&P
-breathy voice is from incomplete closure of vocal cords, can be unilateral vocal cord paralysis
-strained voice is from narrowing of cords such as a mass
-hoarseness onger than 3-4 weeks  ENT referral
-hx of smoked or chewed tobacco  risk of squamous cell carcinoma in upper airway, alcohol is risk factor as well
-blood sputum or oral secretions is a red flag for malignancy

Page 16 of 77
Pathophys
-superior laryngeal nerve (vagus): sensory  supraglottis, motor  inferior constrictor, cricothyroid muscles
-recurrent laryngeal nerve (vagus): sensory  glottis and subglottis, motor  all intrinsic laryngeal muscles except cricothyroid
-HPV 6 and 11 can cause respiratory papillomatosis, benign tumors, rarely give rise to laryngeal carcinoma

Workup
-indirect laryngoscopy in ENT office, direct laryngoscopy is under anesthesia
-CXR if laryngeal cancer is suspected
-CT of neck if concerned about mets to cervical lymph nodes

Management
-stage I and II  surgery or radiation
-stage III and IV  surgery + radiation, chemo and radiation or all 3, may need neck lymph node dissection
-if known cancer, be careful for increasing SOB  may need tracheostomy

Chapter 13: lump on neck


Differential

Page 17 of 77
H&P
-general risk factors for head and neck cancers: alcohol/tobacco, male, >40yo, poor dental hygiene, radiation, African American
-specific cancer risk factors:
-HPV  oropharyngeal cancers
-EBV  nasopharyngeal cancer, Burkitt’s lymphoma
-Chinese  nasopharyngeal cancer
-GERD  laryngeal cancer
-plummer-vinson syndrome (glossitis, cervical dysphagia, Fe deficient anemia, esophageal webs)  pharyngeal and upper
esophageal cancer
-premalignant lesions: leukoplakia, erythroplakia
-symptoms: otalgia, dysphagia, odynophagia, dysphonia, dyspnea, stridor, hemoptysis
-pleomorphic adenoma is most common benign salivary gland tumor, then papillary cystadenoma (warthin’s) of salivary gland
-mucoepidermoid carcinoma most common malignant salivary gland tumor, adenoid cystic carcinoma second most common

Etiology
-Virchow’s node is enlarged left supraclavicular node, where the cisterna chyli (dilated lymph sac at end of thoracic duct) empties into
subclavian vein, suggests metastatic lung or GI malignancy
-most common sites of head/neck cancer is oral cavity, larynx, and pharynx
-malignant neck masses represent spread of cancer via lymphatics until proven otherwise, look for primary cancer
-most common is squamous cell carcinoma in head and neck cancer

Workup
-can observe if present for <3 weeks and not causing any symptoms
-can do flexible nasopharyngoscopy
-should order CBC, BMP, coags, LFTs, TSH
-CT of head and neck (whole body PET/CT to find primary cancer is controversial)
-CXR
-FNA if solitary mass suspicious for metastatic lymph node
-endoscopy (laryngoscopy, esophagoscopy, and bronchoscopy) with biopsy if FNA is positive for metastatic disease
-can do open dissection if all above workup confirms squamous cell carcinoma

Trouble Areas
-missing life-threatening signs in advanced stages of cancer such as stridor, dyspnea, bleeding from mouth or sputum

Chapter 14: hearing loss, ear fullness, tinnitus in child  OME


Differential in Child

Differential for hearing loss in adult

Page 18 of 77
H&P
-peak ages of OME between 2 and 6yo (Eustachian tube becomes more adult like after 6yo)
-risk factors for OME: male, African American, smoke exposure, low birth weight, young maternal age, lower socioeconomic status,
shorter duration or no breast feeding, supine feeding position
-especially suspect if child presents with regression or delay in language milestones
-otalgia with fever, leukocytosis and bulging erythematous TM  AOM
-don’t forget could be foreign body in unilateral ear
-otorrhea is concerning for middle ear disease with TM perforation
-OME main symptom and sign is air-fluid level in middle ear  conductive hearing loss

Pathophys
-sudden deafness  HSV or zoster, most will regain hearing in 2 weeks, give high dose empiric steroids with antivirals
-OME mainly caused by AOM or Eustachian tube dysfunction
-OME in adult should raise flag for cancer obstruction the Eustachian tube, refer to ENT
-main bugs are strep pneumo, H flu, Moraxella (same as AOM) can also have pseudomonas if otitis externa
-most significant complication from OME is conductive hearing loss, also TM scarring and damage (tympanosclerosis = hyaline and
calcium deposition within TM)
-long-term complications: permanent hearing loss, ruptured TM, mastoiditis, temporal bone osteomyelitis, meningitis, brain
abscess

Workup
-pneumatic otoscopy when not obvious on otoscopy
-conductive vs sensorineural hearing loss
-Weber: if sensorineural loss would hear louder in unaffected ear, conductive loss hear louder in affected ear
-Rinne (normal is air > bone): if negative means bone > air

Management
-AOM  abx (amoxicillin with or without clavulanate)
-OME usually will spontaneously resolve in 3 months, observe during this period
-if mild hearing loss should optimize listening/learning environment during this time
-auto-inflation to try to reopen the Eustachian tube
-no indication for antihistamines, decongestants
-steroids with or without abx accelerate short term resolution of OME
-tympanostomy tube for middle ear drainage/ventilation, indications:
-bilateral OME with hearing impairment
-OME with symptoms (vestibular problems, poor school performance, behavioral problems, otalgia, reduced quality of life)
-recurrent AOM with OME
-at risk children with OME (baseline sensory, physical, cognitive factors)
-in adults  ominous for something blocking Eustachian tube, can get nasopharyngoscopy, refer to ENT

Page 19 of 77
PART VI: HEPATOPANCREATICOBILIARY

Chapter 15: RUQ pain  acute cholecystitis


Differential

H&P
-risk factors: female, pregnancy, OCPs (estrogen leads to higher cholesterol in bile), obesity, high fat diet, hereditary (higher in
Hispanics, Pima Indians), Crohn’s disease with terminal ileal resection
-symptomatic cholelithiasis vs acute cholecystitis
-acute cholecystitis: unremitting RUQ pain >6hours, n/v, murphy’s, fever, tachy, elevated WBC, US showing gallbladder wall
thicking, stones, pericholecystic fluid

Pathophys
-pain after fatty foods because triggers release of CCK which contracts gallbladder
-scapular pain because same dermatome with gallbladder via phrenic nerve
-fever, tacchy more signs of severe biliary disease acute cystitis vs cholangitis
-bugs in bile: E coli, bacteroides, klebsiella, Enterobacter, enterococcus, pseudomonas
-two types of gallstones cholesterol or pigmented
-cholesterol: concentration of cholesterol in bile exceeds its solubility in bile salts and lecithin, precipitates out
-pigmented (black or brown): calcium bilirubinate gives color, associated with hemolytic disease, unconjugated bilirubin
increases
-gallstone can cause:
-symptomatic cholelithiasis
-acute cholecystitis
-choledocholithiasis
-cholangitis
-gallstone pancreatitis
-gallstone ileus
-Mirizzi’s syndrome: gallstone in cystic duct so big that it compresses common hepatic duct

Workup
-RUQ US: look for gallbladder wall thickening >4mm and pericholecystic fluid, also diameter of common bile duct (normal 4-5mm)
-can see emphysematous cholecystitis (air in gallbladder wall)  infected with gas forming organisms (clostridium), can
progress to gallbladder perforation, abscess, sepsis, or death, common in older men with diabetes
-LFTs, alk phos, GGT, amylase/lipase
-if RUQ US negative and still high suspicion  HIDA scan

Management

Page 20 of 77
-NPO, MIVF, IV abx, lap chole within 48hours if acute cholecystitis
-common bile duct injury is a complication
-if <50% is involved can repair
-if >50% can bring up jejunum to common bile duct
-if stricture results risk of recurrent cholangitis, cirrhosis, liver failure
-if common bile duct injury is postop detection can come with abdominal pain, bloating, elevated LFTs
-get US or CT to look for fluid collection  blood or bile leak
-if bile leak can do ERCP to see if ductal injury exists
-if cystic duct stump leak, stent into sphincter of Oddi
-if common bile duct or hepatic duct injury may need to bring up jejunum
-RUQ weeks after cholecystectomy  postcholecystectomy syndrome, get CBC, LFTs, RUQ US, can get ERCP if showing ongoing
pathology in biliary tree

Trouble areas
-failing to recognize gangrenous cholecystitis  severe sepsis and gallbladder perforation
-severe unrelenting abdominal pain, high fever, tacchy, markedly elevated WBC, hyponatremia
-more common in men and diabetics
-lap chole
-failing to recognize cholangitis
-charcot’s triad: RUQ pain, jaundice, fever
-Reynold’s pentad: above + hypotension, AMS
-immediate ERCP (or PTC with drain)

Chapter 16: RUQ pain, fever, n/v  acute cholangitis


Differential

-SIRS >2 of below


T >100.4 or <96.8
HR >90
RR >20 or PaCO2 <32
WBC >12

H&P
-gallstones most common cause of cholangitis
-also bile duct strictures, parasites (ascaris or clonorchis liver fluke), ERCP, or biliary stents/drains
-will not have pale stools if acute cholangitis
-charcot’s triad: RUQ pain, fever, jaundice
-reynold’s pentad (implies septic shock): above + hypotension, AMS

Pathophys
-gallstone, bacteria in bile
-untreated cholangitis  severe sepsis, hepatic microabscesses, death

Page 21 of 77
Workup
-hepatic vs posthepatic causes of jaundice
-hepatic causes, such as hepatitis, will show elevated LFTs, AST/ALT out of proportion to alk phos increase, usually
nonsurgical management
-post-hepatic causes, such as biliary obstruction, show rise of alk phos with GGT, surgical management
-RUQ US

Management
-once SIRS is diagnosed  aggressive IV fluids
-admit to ICU
-2 blood cultures then IV broad spectrum abx
-if responds/stable after fluids, abx, NPO  ERCP same admission, elective lap chole if still has gallbladder
-if still unstable after fluids, abx, NPO  emergent ERCP, can repeat ERCP or PTC if still symptomatic, elective lap chole

Trouble
-cholangitis patient also with history of bloody diarrhea  IBD with primary sclerosing cholangitis
-ERCP will show “pearls on string”

Chapter 17: severe epigastric pain radiating to back pancreatitis


Differential

H&P
-Grey Turner’s – blue/black discoloration of flanks  retroperitoneal hemorrhage
-Cullen’s – blue/red discoloration at umbilicus  digested blood products in retroperitoneum

Pathophys
-inappropriate activation of pancreatic enzymes  peripancreatic inflammation
-GET SMASHED
-Gallstones (40%)
-Ethanol (30%)
-Tumors
-Scorpion stings
-Mycoplasma or mumps
-Autoimmune
-Surgery or Trauma
-HLD or hypercalcemia
-Embolic/ischemic
-Drugs/toxins (diuretics:furosemide, thiazides, IBD/immuno: sulfasalazine, azathioprine, seizures: valproic acid, HIV:
didanosine, penamidine)
Page 22 of 77
-3 phases of acute pancreatitis
-premature activation of trypsin
-intrapancreatic inflammation
-extrapancreatic inflammation
-mild pancreatitis: absence of multi-organ failure and local/systemic complications, resolves in 2-5 days
-severe pancreatitis: systemic complications (organ failure) or local pancreatic complications such as pseudocyst, abscess, necrosis
-organ failure
-shock – SBP <90
-PaO2 <60
-Crt >2
-GI bleeding >500mL/day
-hypotension from inflammation and cytokine storm, fluid leaking into retroperitoneal space
-can cause pleural effusion, ARDS
-mortality within first week is usually due to multiorgan failure 2/2 SIRS, after 1 week usually due to sepsis 2/2 pancreatic necrosis or
abscess

Workup
-amylase/lipase, LFTs, CBC, BMP, lipid panel
-RUQ US to r/o gallstone
-KUB will show dilated loops of bowel (sentinel loop in LUQ) and colon cutoff sign at splenic flexure, both due to local ileus from
pancreatic inflammation
-CXR will show left sided pleural effusion
-CT scan if patient not improving after several days to find pancreatic pseudocyst/necrosis/abscess

Management
-NPO, IVF
-morphine vs meperidine, meperidine does not cause contraction of sphincter of Oddi but increases risk of seizures
-if hospitalized, in 3 weeks gets fever, leukocytosis  CT scan to find abscess or necrotic tissue
-abx, surgical debridement
-if patient comes back in 4 weeks with persistent abd pain, epigastric mass, elevated amylase  CT scan for pseudocyst
-if <6cm, observation with serial CT scans
-if >6cm or symptomatic, surgical drainage
-chronic pancreatitis  can develop DM, steatorrhea, chronic pain

Trouble
-don’t forget hypercalcemia causes pancreatitis (maybe on thiazides) but also then pancreatitis can cause hypocalcemia
-can get pseudohyponatremia in HLD pancreatitis, true Na is normal
-if NPO >7 days get feeding tube past ligament of Treitz to avoid activating pancreas

Chapter 18: painless jaundice  pancreatic cancer


Differential for jaundice

Page 23 of 77
H&P
-Courvoisier’s sign  palpable RUQ mass (nontender, enlarged gallbladder) signifying obstruction, usually of distal common bile duct
-new onset T2DM in older patient is suggestive of tumor in pancreatic body
-painful jaundice implies acute biliary obstruction usually from gallstone
-painless jaundice is a more insidious obstruction
-risk factors for pancreatic cancer: chronic pancreatitis, tobacco, high fat diet, male, fam hx, or if UC can get primary sclerosing
cholangitis
-risk factor for bile duct cancer: choledocholithiasis especially from parasites
-risk factor for gallbladder carcinoma: long standing gallstone disease
-sister mary joseph nodule  metastatic abdominal/pelvic malignancy, mostly GI malignancies
-Blumer’s shelf  step off in rectal exam, signifying metastatic disease to pouch of Douglas, usually mets of lung, pancreas, stomach
cancer

Pathophys
-breakdown of RBCs, bilirubin is unconjugated bound to albumin then conjugated in liver and into the intestine, converted to
urobilinogen by bacteria, reabsorbed into circulation again, converted to urobilin and into urine, remaining urobilinogen in intestine is
converted to stercobilin and into stool
-pale stools with biliary obstruction because no stercobilin to give stool its color

Workup
-T bili, direct bili, LFTs, alk phos and GGT
-painful jaundice  get RUQ US
-painless jaundice  triple phase CTAP (arterial, early venous, late venous phase to detect pancreatic masses)
-EUS as well
-not really MRCP/ERCP

Management
-resectability determined by triple-phase CT scan with or without EUS
-nonresectable if invasion into SMA, celiac, or hepatic arteries, or if disease is metastatic
-no need to prophylactically decompress via ERCP/stenting
Page 24 of 77
-neoadjuvant therapy for borderline resectable patients
-whipple complications
-delayed gastric emptying, try metoclopramide
-pancreatic leak/fistula, biliary leak/fistula, hemorrhage

Trouble
-malignancy so advanced that biliary duct is obstructed, no bile, no vit K absorption in GI tract, prolonged INR  can give IV vit K or
FFP if actively bleeding

PART VII: LOWER GI

Chapter 19: BRBPR  diverticulosis


Differential for lower GI bleeding

H&P
Page 25 of 77
-older patients >50yo LGIB is more from diverticulosis, angiodysplasia, malignancy
-younger patients <50yo more likely infectious, hemorrhoids, fissures, IBD
-diverticulosis bleeds are arterial and present more acutely with large amounts of blood
-angiodysplasia and cancer are more chronic bleeds, presents more with anemia and dark stools
-right colonic diverticulosis more likely to bleed, left ones are more likely to get infected
-hx of pelvic radiation on aortic surgery repair; radiation damages rectal mucosa causing radiation proctitis, aortic surgery can have
erosion of aortic graft into duodenum causing aortoduodenal fistula
-if liver disease can cuase coagulopathy and varices in rectal veins if cirrhotic
-abdominal tenderness suggestive of colitis from IBD, ischemic colitis, infectious diarrhea
-ischemic colitis is typically left sided abdominal pain, bloody diarrhea in elderly patient who is volume down
-bleed after colonoscopic biopsy or polypectomy several weeks before  endoscopic injection of vasoconstrictive agents (epi), or
cauterization, or clipping

Etiology/Pathophys
-diverticulum: false diverticulum, mucosa and submucosa herniates through muscular layer of intestinal wall, most common in sigmoid
colon, diverticulosis bleeding is arterial
-angiodysplasia: thin, weak, dilated vessels in GI tract, most commonly in cecum and right colon in the elderly, usually small bleeds
with anemia, venous bleeding
-ischemic colitis: watershed areas, splenic flexure between SMA and IMA, hypoperfusion state, can be precipitated by dehydration,
heart failure, shock, CV surgery, usually will resolve spontaneously with fluids/supportive care, usually only affects mucosa
-acute mesenteric ischemia: from arterial embolus/thrombosis, causes bowel necrosis, usually in SMA region (small bowel to
mid-transverse colon), and is often transmural, CT scan can show pneumatosis, bowel wall thickening, even occlusion of SMA

Management
-LGIB: 2 large bore IVs, type and cross, CBC, BMP, INR, crystalloid or pRBCs
-NG tube to r/o upper GI bleed, if returns with blood then UGIB  EGD, if returns with bile then LGIB
-colonoscopy
-if cannot find source of bleed, diagnostic arteriography (also therapeutic, can embolize) if fast bleed or tagged RBC scan using
technetium-99 if slow bleed
-if think bleed is upper GI, can do Meckel’s scan, capsule endoscopy, enteroscopy
-if bleeding still has not stopped can do emergent laparotomy with total colectomy with end ileostomy if bleed is somewhere in colon

Page 26 of 77
Chapter 20: RLQ pain  acute appendicitis
Differential

Differential (women)

Differential (children)

Page 27 of 77
H&P
-absent bowel sounds  paralytic ileus 2/2 inflamed/infected bowel
-Rovsing’s sign  RLQ pain with LLQ palpation, from stretching LLQ abdominal wall triggering pain in RLQ partietal peritoneum
-Psoas sign  RLQ pain with passive right leg extension
-Obturator sign  RLQ pain with internal rotation of hip
-McBurney’s point  1/3 from anterior iliac spine to umbilicus

Pathophys
-periumbilical pain to RLQ pain  autonomic nerves to visceral peritoneum (poorly localized, dull pain) first in periumbilical region
then as inflammation progresses somatic innervation to parietal peritoneum, somatic spinal nerves supply skin causing hyperesthesia of
skin
-foregut pain (esophagus to distal duodenum) is usually perceived in epigastrium
-midgut pain (ligament of Treitz to 2/3 of transverse colon) is usually perceived as periumbilical
-hindgut pain (left colon and rectum) is usually perceived as hypogastrium/suprapubic
-can have closed loop obstruction in appendicitis (since it is a blind loop and if one end is obstructed by fecalith), begins to dilate/distend
until venous pressure >arterial then ischemia/infarction, can perforate
-usually fecalith is most common cause, in kids can be lymphoid hyperplasia

Workup
-CBC for leukocytosis with left shift (immature white cells or band cells = bandemia), CRP, get beta-HCG in women
-if diagnosis is classic in adult male, no imaging is needed
-if equivocal, US especially in women and children, CT scan for men and nonpregnant women

Management
-lap appy
-preoperative abx
-abx for 3-5 days if perforated/gangrenous abx
-if during lap appy find that appendix looks normal, remove anyways

Trouble
-any peritonitis can cause inflammation of serosa of appendix  peri-appendicitis
-pseudoappendicitis from Yersinia entercolitica, also with bloody diarrhea, can give doxy
-women:
-missing ovarian torsion (can get US or CT scan): surgical emergency
-missing PID (can get US or CT scan): usually with lower abdominal pain, bilateral, history of foul smelling discharge
-missing ruptured ectopic: get a beta-HCG, surgical emergency
-missing appendicitis in pregnancy: get US or MRI
-children: more prone to early rupture
-elderly: more prone to rupture, don’t miss perforated colon cancer
-if appendicitis with >5day RLQ pain  already perforated
-either infection can spread to peritonitis
-or localized abscess  will show localized pain/tenderness, get CT scan, can drain percutaneously, start IV abx

Page 28 of 77
Chapter 21: pencil-thin stools with constipation  left colon cancer
Differential for change in bowel habitus

Screening
-age 50 to 75
-colonoscopy every 10 years
-flex sig every 5 years +FOBT q3years
-FOBT every year
-if first degree relative with colon cancer start screening at age 40 or 10years before age of onset in relative, then screen every 5 years

H&P
-risk factors: age, AA, IBD, fam hx, low fiber/high fat diet, obese, smoking, alcohol, radiation to abdomen
-men: most common prostate, lung, colon, mortality is lung, prostate, colon
-women: most common breast, lung, colon, mortality is lung, breast, colon
-majority of colon cancer is left sided: change in bowel habitus, symptoms of obstruction
-right sided is usually anemic, can be melenic stools
-do a DRE

Pathophys
-nonneoplastic polyps:
-hyperplastic – most common type, small <5mm and smooth
-juvenile (hamartomatous)– occur mostly in children, rounded smooth, sometimes have a stalk, usually solitary rectal polyp
-neoplastic polyps – adenoma-carcinoma sequence (chromosome instability pathway)  loss of APC tumor suppressor, KRAS
mutation, loss of p53 tumor suppressor gene, sequence takes about 10 years (screening colonoscopy q10yrs)
-juvenile polyposis syndrome – large number of hamartomatous polyps in stomach and colon in children
-Peutz-Jeghers syndrome – multiple hamartomatous polyps alongside mucocutaneous hyperpigmentation on lips+genitals
-adenomas
-tubular – most common, usually in rectosigmoid
-villous – less common, larger and in elderly, usually rectum, usually sessile/cauliflower-like, high malignant potential
-tubulovillous – both tubular and villous features, can be pedunculated or sessile
-sessile serrated – flat, broad polyps that can look like hyperplastic polyps, high malignant potential, “saw-tooth” appearance
under microscope
-metastatic sites – regional to lymph nodes, distant to liver (mesenteric veins draining to portal system), also these spread
hematogenously
-rectal cancers may have atypical locations of mets because they drain into both portal and systemic circulations, can go to
lungs, inguinal lymph nodes, spine/brain
-heritable conditions

Page 29 of 77
-HNPCC/Lynch – autosomal dominant, colorectal, endometrial, ovarian, stomach, breast, small bowel, pancreatic, kidney, bile
duct cancers; colon cancers are from adenomas but usually flat rather than polypoid, more likely malignant, more rapid
-Lynch 1 – heterozygous mutation that results in more right sided colonic cancers in 4th decade of life
-Lynch 2 – MLH1 gene mutation that results in more extracolonic cancers
-3, 2, 1, 1 rule: 3 or more relatives with verified cancers in colon/endometrium/small intestine/pelvis, 2 or more
successive generations affected, 1 or more relatives diagnosed <50yo, 1 is 1 st degree relative
-FAP – autosomal dominant, APC gene, polyps begin mid-teens, prophylactic colectomy otherwise 100% ill develop cancer
by 4th or 5th decade of life
-if 1st degree family member with FAP, then should begin colonoscopy screening at age 10
-Gardner’s syndrome – autosomal dominant, variable penetrance, osteomas and colonic polyps, if untreated then colon cancer
by 4th or 5th decade of life
-Turcot syndrome – café au lait spots, malignant CNS tumors (GBM), colon polyps that progress to cancer
-synchronous tumor: 2nd primary cancer
-metachronous tumor: primary cancer that develops elsewhere in colon >6mo after primary resection, distinguish from recurrent cancer

Workup
-colonoscopy
-CEA levels
-LFTs
-CT chest, abd/pelvis to look for mets
-if rectal cancer, get transrectal US and MRI to for sphincter-preserving surgery

Management
-polyps should be removed with polypectomy and surveyed 1-5 years after
-right-sided colon cancer  right colectomy with ligation of ileocolic artery
-transverse colon cancer  transvers colectomy or extended right colectomy with ligation of ileocolic and middle colics
-descending colon cancer  left colectomy with ligation of IMA
-sigmoid colon cancer  left colectomy with ligation of IMA
-if rectal cancer  >3cm from dentate line can do LAR and anastomose proximal colon with distal rectum
 if <3cm from dentate line, abdominoperineal resection to remove entire distal rectum and anus with sphincters, leaving
permanent colostomy
-remove minimum of 12 lymph nodes
-neoadjuvant therapy for rectal cancer for better operation, not really for colon cancer
-radiation for rectal cancer not colon cancer, since cannot radiate entire abdomen/pelvis
-colon surgery complications: injury to ureters (right/left colectomy), duodenum (right colectomy), spleen (left colectomy) or
anastomotic leak
-recommendation is to follow CEA q3months for 3 years after surgery

Chapter 22: chronic constipation with severe abdominal pain  LBO


Differential

Page 30 of 77
H&P
-constipation implies infrequent stools with hard stools, obstipation implies complete absence of gas or stool in rectum
-check for irreducible hernias
-hx of neurologic/psych diseases because drugs for those can affect GI motility and predispose
-Ogilvie’s syndrome: hospitalized patient or post-op setting with similar symptoms to LBO

Etiology/Pathophys
-most common cause of LBO is colon cancer (usually left sided), diverticulitis, then volvulus
-cecum volvulus is more congenital from malrotation, sigmoid volvulus is acquired
-complicated volvulus means there is bowel ischemia

Workup
-CBC, lactate, BMP
-KUB (supine and upright) and upright CXR
-CT abdomen with PO and IV contrast if LBO or volvulus unclear
-contrast enema could show bird’s beak or ace of spades sign

Management
-LBO:
-IV fluids
-foley
-NGT if symptomatic
-common in pregnancy, detorsion but can resect after delivery
-sigmoid volvulus
-endoscopy to untwist if uncomplicated, or contrast enema, then semi-elective sigmoid resection since recurrence is high
-if complicated, resect and colostomy
-cecal volvulus
-surgery is first line  right colectomy with anastomosis
-Ogilvie’s syndrome can progress to dilation and perforation, give neostigmine and decompress

Chapter 23: LLQ pain and fever  diverticulitis


Differential

H&P
-risk factors: obesity, diet low in fiber or hat in fat/red meat, age
-diverticulitis is a clinical diagnosis

Etiology/Pathophys
-diverticula (false diverticulum) usually in sigmoid colon, those in left/sigmoid colon are more likely to be infected, diverticula in right
more likely to bleed
-diverticula rare/never in rectum since taenia coli bands coalesce into band around rectum
Page 31 of 77
-complications of diverticulitis: abscess, perforation, fistula, stricture, LBO
-colovesical fistula from diverticulitis (most common), colon cancer, IBD, bladder cancer, radiation, trauma
-present with fecaluria, pneumaturia, recurrent UTIs with multiple enteric organsisms/anaerobes

Workup
-CTAP
-barium enema and colonoscopy contraindicated because of inflamed colon, risk of perforation

Management
-SIRS
-T >100.4 or <96.8
-HR >90bpm
-RR >20 or PaCO2 <32mmHg
-WBC >12,000 or <4000 or >10% bands
-if uncomplicated diverticulitis with no SIRS, d/c home with PO abx 7-10days and clears
-if uncomplicated diverticulitis with SIRS, admit, NPO, IV abx
-if no improvement, colectomy
-colonoscopy after inflammation subsided to rule out IBD, colon cancer
-if complicated diverticulitis
-if peritonitis, perforation  emergent surgery, colectomy, colostomy can be reversed when inflammation resolves
-LBO  urgent colectomy with colostomy
-if abscess <4cm with no peritoneal signs  bowel rest, abx
-if abscess >4cm can do CT guided drainage
-if with colovesical fistula  IV abx, resect segment of colon, bladder repair

PART VIII: NEUROSURGERY

Chapter 24: Neck pain and paralysis after trauma  burst fracture dislocation, anterior spinal artery syndrome
Differential

H&P
-most common cervical spinal levels involved after trauma is C2, C6, then C7
-most common level of subluxation injury is C5/C6 interspace, area of greatest flexion/extension
-dermatomes; C5 to T1 upper extremities
-shoulders C4
-nipples T4
-umbilicus T10
-knees L4
-perianal region S4/S5
-complete spinal injury no motor or sensory below level of injury, incomplete injury still have some residual function below level of
injury
-if injury at or above C3, diaphragmatic paralysis and all 4 limbs
-pathology of nerve root (radiculopathy): burning, tingling pain down a limb, LMN signs
-pathology of spine (myelopathy): neck pain that radiates to shoulders/occiput, bilateral UMN signs
Page 32 of 77
Pathophys
-spinal shock – flaccid paralysis below level of injury with loss of all reflexes, including urinary/rectal tone
-if there is sacral sparing (intact sphincter or perianal sensation) spinal shock has better prognosis
-atlanto-occipital dislocation is most unstable/dangerous, patients with Trisomy 21 at risk, should be screened before sports
-incomplete spinal injuries
-brown-sequard syndrome: hemisection of spinal cord, usually from penetrating trauma such as gunshot or stab wounds,
causing injury to corticospinal tract, posterior columns, spinothalamic tracts, best prognosis of the 3
-anterior cord syndrome: damage to anterior 2/3 of spinal cord due to severe flexion injury, corticospinal and lateral
spinothalamic tracts affected bilaterally, worst prognosis of the 3
-central cord syndrome: damage to central portion associated with severe extension injury, often elderly with pre-existing
cervical stenosis, distal lower extremities spared

Workup
-AP, lateral, open-mouth cervical Xray
-cervical spine Xrays in trauma patients unless have
-neurologic deficit
-spinal tenderness
-AMS
-intoxicated
-distracting injury
-CT if pathology on Xray
-MRI can identify injury to spinal cord itself in patients with neurological deficits
-only imaging modality for SCIWORA (spinal cord injury without radiographic abnormalities)

Management
-if cervical spine injury
-cervical collar, tongs/halo traction device
-IV fluids/pressors for BP
-foley
-VTE ppx
-if neurogenic shock  crystalloid NS, can use pressors of dopamine, phenylephrine, if bradycardia use atropine or dopamine
-emergent surgery with spinal cord injury if
-unstable vertebral fracture
-nonreducible spinal cord compression with deficit
-ligamentous injury with facet instability
-lower threshold for surgery in incomplete vs complete spinal cord injury because incomplete more likely to maintain/regain function

Chapter 25: loss of consciousness after head trauma  TBI, epidural hematoma
Differential for TBI with GCS <8

Page 33 of 77
H&P
-traumatic brain injury – period of loss of consciousness loss of memory before/after accident, AMS, focal neurologic deficit
-GCS <8 is considered coma, GCS 9-12 is moderate injury

-GCS can be altered by alcohol, drugs, sedatives, severe hypoxia, severe shock, severe hypothermia
-Raccoon eyes  bilateral periorbital ecchymosis, battle’s sign  retroauricular ecchymosis, indicators for basilar skull fracture
-paralysis contralateral to lesion, abnormal pupillary findings ipsilateral to lesion
-can have paralysis and pupillary findings on same side (Kernohan syndrome) if contralateral cerebral peduncle against contralateral
tentorial incisure, a false localizing sign
-decorticate posturing  flexion in upper extremities and extension in lower extremities to painful stimuli, slightly better prognosis than
decerebrate
-decerebrate posturing  extension of upper and lower extremities

Pathophys
-concussion = mild TBI, temporarily alters brain functions in memory, balance, coordination, concentration, can be associated with
headaches, dizziness, confusion, irritability; loss of consciousness is not required to establish diagnosis
-uncal herniation  space occupying lesion above tentorium that displaces uncus of temporal lobe over tentorial incisure hitting the
ipsilateral CNIII and ipsilateral cerebral peduncle causing ipsilateral blown pupil and contralateral paralysis
-lucid interval classically seen with epidural hematomas, initial loss of consciousness results from disruption of brainstem arousal centers
(RAS), second loss of consciousness is from hematoma and mass effect
-cerebral perfusion pressure = MAP – intracranial pressure
-most potent vasodilator is blood CO2 level, mild hyperventilation lowers CO2 and can provide temporary therapeutic benefit for
elevated ICP
-Cushing’s triad/reflex = HTN, bradycardia, irregular RR  physiologic response to increased ICP
-coup = injury to brain tissue below the skull at point of impact
-contrecoup = injury on opposite side of brain from impact, so can have contusion on opposite sides of brain

Management
-GCS <8 should intubate
-CT head w/o contrast
-GCS 9-12 and below should get CT head w/o contrast
-patients with mild brain injury and certain risk factors (anticoagulated, alcohol abuse, elderly) also get CT scan
-succinylcholine and ketamine increase ICP and are contraindicated in TBI, use rocuronium and etomidate
-ICP monitoring if: (normal ICP 5-15mmHg)
-GCS <8 with abnormal head CT
-GCS <8 with normal head CT and any 2 of: hypotension, posturing, >40yo
-when neuro exam is not appropriate assessed and with suspicion of high ICP
-increased ICP treatments:
-ventriculostomy tube
-mild hyperventilation if CT showing or signs of increased ICP, goal of PaCO2 30-35
-mannitol
-can elevate head of bed, paralyze patient, hypothermia, barbiturate coma, or craniectomy
-craniotomy if acute subdural/epidural hematoma with midline shift >10mm, hematoma >5mm, ICP >20mmHg
-nonoperative management
-urgent repeat head CT if new neurologic signs, continued vomiting, worsening headache, loss of >2 on GCS, or increased ICP
-brain death criteria
-GCS of 3
-euthermic >32.2C
Page 34 of 77
-PaO2 >90
-systolic BP >100
-no sedation/paralytic
-absent brainstem reflexes (corneal, gag, ocular)
-no response to deep central pain
-agreement of 2 physicians
-if all conditions above are met then apnea test  d/c from ventilator and observed for respiratory effort, if no spontaneous
respirations when PaCO2 >60 then brain death can be pronounced
-in children, head CT if
-GCS <14, suspected basilar fracture, palpable skull fracture, or AMS
-individualized basis other factors include loss of consciousness >5s, scalp hematoma, severe mechanism of injury, history of
vomiting, severe headache, child not acting normal per parent

PART IX: ORTHOPEDIC

Chapter 26: multiple extremity injuries after MVC


-radial nerve injury is common in middle or distal third of humerus fractures, gives off innervation to triceps then goes to innervate
distally
-injury can have wristdrop, loss of MP extension, loss of sensory over dorsum of hand
-can still extend fingers at IP joints since ulnar and median nerves still intact

H&P
-evaluate 4 function components in extremities after trauma: nerves, vessels, bones, and soft tissues
-injury to 3 out of 4 of the above constitutes “mangled extremity”
-open fractures have higher risk of infection, and heals slower
-fat embolism: typically 24-72 hours after trauma  respiratory symptoms, neurological changes, reddish-brown petechial rash (rash is
pathognomonic for fat embolism syndrome)
-compartment syndrome: pain out of proportion, pressure (swollen/tense), paresthesia, pulselessness, poikilothermia, paralysis 
surgical emergency
-femur fracture, don’t miss femoral neck fracture because can cause avascular necrosis (irreversible hip joint dysfunction)

Pathophys
-nerve injury
-neuropraxia – minimal injury, just myelin, not axon or nerve sheath, temporary nerve conduction block, loss of motor and
sensory function but not autonomic, full recovery expected in hours to months
-axonotmesis – myelin plus axon disrupted, nerve sheath intact, Wallerian degeneration with motor, sensory, and autonomic
paralysis, recovery is incomplete in weeks to months (axon sprouts within nerve sheath)
-neurotmesis – myeline, axon, and nerve sheath damaged, Wallerian degeneration, recovery variable and incomplete, usually
requires surgery or results in complete paralysis
-injured axon typically recovers 1mm/day
-layers of nerve sheath: endoneurium, perineurium, epineurium
-classic nerve injuries with fractures:

Page 35 of 77
Workup
-radiographically
-floating knee = ipsilateral femur and tibia fracture
Management
-grading of open fractures (Gustilo-Anderson system)  increased grade correlates with risk of infection, nonunion, amputation

-most important determinant of severity for open fractures is the energy imparted to limb during trauma
-all open fractures should receive abx, take to OR asap, usually within 6 hours of injury
-grade I and II at risk for gram(+) infection so 1 st gen cephalosporin
-grade III should get cephalosporin + aminoglycoside
-continue abx 24hours if wound is closed, if closure is delayed keep abx until 24 hours after final closure
-add penicillin or its equivalent if trauma from farm accidents/soil-contaminated wounds to cover for Clostridium perfringens
(gangrene)
-if dirt and soil contaminates wound: clostridium tetani, give tetanus toxoid/tetanus IG
-reduction = returning parts back to normal position, can either do through incision (open reduction) or external manipulation manually
(closed)
-fixation = holding bone in place with instrumentation, can either do with internal fixation, completely inside body, will be permanently
in place vs external fixation which needs to be removed at some point for risk of infection
-long bone fractures (femur, humerus) risk for fat embolish syndrome, blood loss, compartment syndrome, hemorrhagic shock
-pelvic fractures are greatest risk for hemorrhagic shock
-femur fracture repair within 2-12 hours of injury with intramedullary nailing (internal fixation) plus/minus external fixation
-femur fracture in children 3-5years can be done with spica casting, no need for fixation (be careful for child abuse or
osteogenesis imperfecta)
-fat embolism syndrome: ventilator support with high PEEP and early stabilization of fractures
-early stabilization within 24hours of long bone fractures to prevent fat emboli
-postop: can have infection, iatrogenic nerve/vessel injury, nonunion (failure to heal), malunion (healing in improper position), or
instrumentation failure

Trouble
-missing another fracture, pelvic fractures can lead to hemorrhagic shock
-compartment syndrome (lower limb and forearm more at risk, then hand and foot second most common)
-rhabdomyolysis with kidney failure if crush injury
-missing neurologic deficit

Chapter 27: Immediate knee swelling after trauma  ACL tear


Differential for acute knee pain

Page 36 of 77
H&P
-knee exam

-history of locking or catching with range of motion of knee can signify mechanical blockage to motion, most common with meniscal
tears or other osteochondral bodies in joint space
-always assess pulses to r/o knee dislocation or fracture
-always assess neuro to see if dislocation, fracture, if need stabilization or surgery
-if recurrent fractures need to consider neoplastic lesion weakening the bone
-if can bear weight, less likely fracture or septic joint

Pathophys
-unhappy triad of knee = MCL, ACL, medial meniscus injuries
-meniscus blood supply is outside to in, tears in outer zone can heal and are repaired, inner tears in “white” zone don’t have direct blood
supply, won’t heal, and are surgically debrided not repaired
-“popping” noise on traumatic injury is most commonly with ACL rupture, can be with meniscal injuries
-PCL injury is most common with car dashboard knee injury since knee is flexed when it hits
-if patient unwilling to range the knee think fracture or septic joint
-if cannot actively fully extend knee but can do it passively then disruption of quadriceps/patellar tendon or fracture of patella/tibial
tubercle

Workup
-xrays for ACL: small fleck of bone avulsed from lateral tibial plateau (segond fracture)
-MRI to confirm
-patella alta and baja corresponds with patellar tendon rupture or quadriceps rupture respectively
-hot, swollen knee without trauma should get arthrocentesis for septic joint, crystals for gout (monosodium urate, negatively birefringent)
or pseudogout (calcium pyrophosphate, positively birefringent), WBC <20 is normal, inflammatory if 200-50,000, >50,000 is infectious

Management
-nonoperative management for ACL tear can be for partial tears from low-energy mechanisms, or if patient is older, less active, or if
patient has osteoarthritis (can’t operate on arthritic knee), or not good surgical candidate
-activity modification, bracing, PT
-acute treatment is RICE
-surgery: arthroscopic reconstruction of ACL (not repair), uses tendon graft, repairing native ligament doesn’t really heal
-meniscal tear is usually nonoperative management, surgery only if persistent/disabling symptoms or with large tears
-postop concerns: infection/septic joint that can damage cartilage and cause arthritis, knee cartilage breakdown with ACL dysfunction,
knee stiffness, graft failure

Page 37 of 77
Chapter 28: Groin pain with limp  SCFE
Differential for groin pain with limp

H&P
-in adults, think tumor or mets, multiple myeloma
-children <5yo, ALL or neuroblastoma
-Legg-Calve-Perthes typically 4-8yo
-SCFE, obese 10-16yo male, more common in AA or Polynesian ethnicities
-usually present with painful limp, refusal to bear weight, pain in groin but can localize to thigh or knee, decreased internal
rotation, held in external rotation
-also associated with endocrine disorders: hypothyroidism, GH abnormalities, renal osteodystrophy, hypopituitarism especially
if patient is <10yo and thin
-psoas abscess will hold in slight hip flexion and internal rotation
-psoas abscess is sign of underlying etiology such as osteomyelitis of spine, PID
-if held in hip flexion and internal rotation after trauma  hip dislocation
-if recent URI could be transient synovitis, more common in children <5yo but can be in adolescents
-hx of travel/wildlife exposure can indicate septic joint, TB, lyme disease
-hx of corticosteroid use can predispose pediatric population to avascular necrosis
-hx of sickle disease in kids

Pathophys
-DDH – abnormal hip development
-Legg-Calve-Perthes disease – unknown pathophys, idiopathic osteonecrosis of femoral head, usually 4-8yo
-SCFE – displacement of metaphysis of femoral neck, usually 10-16yo obese male

Workup
-CBC, BMP, ESR/CRP to r/o infection
Page 38 of 77
-blood cultures, joint aspirates
-Xrays
-MRI

Management
-SCFE – in situ screw fixation
-complications: if screw too long could penetrate hip joint and destroy cartilage, if too short more slippage/dysfunction can
occur, also avascular necrosis in general

Chapter 29: Chronic right hand pain  Carpal tunnel syndrome


Differential

H&P
-median nerve distribution (palmar 3.5 digits), likely carpal tunnel
-Tinel’s sign: percusing over median nerve at carpal tunnel with electrical shock sensation
-Phalen’s test: place dorsal sides of hands against each other for max flexion for 30-60s, positive if patient reports paresthesias
in median nerve distribution
-Durkan’s test: squeezing patient’s wrist over median nerve at tunnel, positive if new/worsening pain/tingling/numbness in
median distribution within 30-60s
-median nerve can also be compressed at elbow (pronator syndrome), or median nerve tumor (schwannoma)
-pronator syndrome in elbow: palmar cutaneous branch of median nerve branches prior to tunnel and innervates skin over
thenar eminence, so will have dysesthesias here if elbow is location of compression
-Spurling’s test for cervical root pathology: extend neck, tilt and turn head laterally to affected side to see if dysesthesias worsen
-thoracic outlet syndrome – compression of lower brachial plexus (ulnar symptoms) or compression of subclavian vessels
-Adson’s test: extension of shoulder with neck turned toward affected side can reproduce symptoms or reduce pulse at wrist
-Wright’s test: abduction/external rotation of shoulder with neck away from affected side, see if produce symptoms
-myelopathy if atypical distribution

Pathophys
-median nerve (C5-T1) innervates thenar muscles and two radial lumbricals, and sensory to palmar 3.5 fingers and dorsal tips
-boundaries of carpal tunnel – carpus dorsally (floor), transverse carpal ligament is roof
-9 tendons and 1 nerve go through carpal tunnel
-4 flexor digitorum superficialis tendons
-4 flexor digitorum profundus tendons
-1 flexor pollicis longus tendon
-median nerve (most superficial)
-risk factors for carpal tunnel syndrome: female, age 40-60, obesity, pregnancy, smoking, activities with sustained wrist flexion,
rheumatoid arthritis, hypothyroidism
-stages of median nerve compression
-stage 1 – sensory symptoms at night (numbness, pain, tingling)

Page 39 of 77
-stage 2 – symptoms during day
-stage 3 – motor weakness and muscle wasting (thenar eminence), where nerve is dying and hypoexcitable in Tinel’s, Phalen’s,
or Durkan’s test
-median nerve laceration at elbow (high median nerve injury) only ring and small fingers form the fist, first three fingers cannot flex
-ulnar claw – extension at MP, flexion at IP, all intrinsic muscles are denervated except for radial two lumbricals
-hypothyroidism is commonly associated with carpal tunnel syndrome, excess glycosaminoglycans and hyaluronic acid deposited in
subcutaneous tissue and median nerve sheath

Workup
-exam and history is diagnostic, can do EMG
-can do MRI and xrays of cervical spine to look for root/cord pathology (CT myelogram if can’t do MRI)

Management
-1st line – splinting in neutral position, activity modification, NSAIDs
-2nd line – local anesthetic and steroid injuection
-failure to respond to conservative treatment or thenar motor involvement  surgery (carpal tunnel release, cutting transverse carpal
ligament)
-complications – injury to median nerve, chronic pain, injury to superficial palmar vascular arch, recurrence
-can injure recurrent motor branch of median nerve if transverse ligament is transected too radially, will cause greatly impaired
function of thumb if nerve is injured
-palmar cutaneous branch lies above transverse carpal tunnel, if cut can lead to painful/difficult to treat neuroma
-acute carpal tunnel syndrome (from fracture or dislocation of wrist) is surgical emergency

Trouble
-missing spinal pathology
-missing treatable neuropathy – thyroid or diabetes
-missing Pancoast lung tumor invading on brachial plexus
-missing brain tumor

PART X: PEDIATRIC

Chapter 30: Full-term male infant with respiratory distress  congenital diaphragmatic hernia
Differential for surgical causes of neonatal respiratory distress

H&P
-grunting and retractions  severe respiratory distress, intubate
-absent breath sounds, displaced heartbeat, absence of prenatal care

Pathophys

Page 40 of 77
-congenital diaphragmatic hernia from failure of septum transversum to completely divide pleural and coelomic cavities during fetal
development, fusion of the diaphgram precursor is usually completed posteriorly by 12 weeks
-85% are on left side, 10% on right side, 5% bilaterally
-pulmonary hypoplasia with muscular hyperplasia of pulmonary arterial tree causes high resistance that does not reverse with infant’s
first breath, causing hypoxemia, acidosis, hypotension which causes pulmonary vasoconstriction
-pulmonary hypoplasia results in decreased gas exchange and carbon dioxide retention
-mortality is related to degree of pulmonary hypoplasia and pulmonary HTN, or any other anomalies

Workup
-OGT/NGT
-CXR
Management
-if unstable  intubate
-OGT to low continuous suction to decompress stomach/small bowel
-admit to NICU
-ventilation senting FiO2 100%, PEEP at 3-5, high-frequency ventilation if still hypoxia/hypercarbia
-NO for pulmonary HTN
-ECMO for severe/refractory cases
-repairing diaphragm will not improve pulmonary function
-if mild case with little or no pulmonary HTN, can do repair in 48-72 hours
-most patients, surgery is delayed for cardiopulmonary support and awaiting lung maturation and reversal of pulmonary HTN

Trouble
-want to intubate ASAP, avoid blow-by or mask ventilation because will inflate GI tract and worsen lung compression
-avoid barotrauma when intubated, may have to do permissive hypercarbia PaCO2 <60 and PaO2 ~60 to avoid
overventilation/barotrauma
Chapter 31: Newborn with Bilious Emesis  duodenal atresia
Differential for neonatal bilious emesis

H&P
-rules out pyloric stenosis because bilious emesis is coming through
-polyhydramnios because fetus cannot swallow
-passage of meconium does not exclude the intestinal obstruction
-most common associated abnormalities for duodenal atresia: down syndrome, annular pancreas, congenital heart disease, malrotation

Pathophys
-duodenal obstruction can be from duodenal atresia (most common), Ladd’s bands, annular pancreas
-embryologic events around 6 weeks of gestation, duodenal atresia when failure of gut to recanalize
-jejunoileal atresias are vascular accidents

Workup
-2 view abdomen (AP and lateral)

Page 41 of 77
- if double bubble sign  duodenal atresia
-if distal bowel gas  UGI contrast  malrotation with or without midgut volvulus
-if distal obstruction (multiple dilated loops of bowel)  contrast enema  intestinal atresia, meconium ileus, or Hirshsprung’s
-if bilious emesis and patient is unstable  suspect malrotation with midgut volvulus, give abx and OR

Management
-IV access, fluid replacement, NGT tube
-if patient becomes unstable should question duodenal atresia diagnosis, prepare for OR and laparotomy, most likely malrotation with
volvulus
-if duodenal atresia, surgery can be delayed until work-up is completed, usually first few days of life
-if very premature can delay few weeks, give TPN, want lung maturation
-surgery is duodenoduodenostomy, if cannot do then duodenojejunostomy
-should also look for associated abnormalities including echo, renal US, spinal radiographs, chromosome abnormalities

Trouble
-inadequate fluid resuscitation before surgery, since duodenal atresia is rarely a surgical emergency
-failure to rule out heart defects prior to surgery
-injury to annular pancreas or ampulla

Chapter 32: Infant with bilious emesis  malrotation with midgut volvulus
Differential

H&P
-associated risk factors include heterotaxy syndrome, duodenal atresia

Workup
-if HDS  xray abd
-if proximal obstruction on xray  UGI contrast series
-if distal obstruction (multiple dilated loops)  get abdominal US to evaluate intussusception
-if infant is hemodynamically unstable with symptoms of acute obstruction no imaging necessary  OR directly

Pathophys
-midgut is from SMA  2nd part of duodenum to 2/3 of transverse colon
-normally: 6th week gestation midgut elongates quickly and grows outside of embryo, between 10-12 weeks the midgut comes back into
abdominal cavity, proximal limb goes behind SMA
-failure of midgut to rotate and fix properly so that ligament of Treitz is right of midline and cecum doesn’t get to RLQ, instead
stays in epigastrium, then attachments form between ascending colon and RUQ which cross the duodenum
-“corkscrew” appearance on UGI contrast series

Management

Page 42 of 77
-Acute midgut volvulus is a surgical emergency
-fluid resuscitation and OR
-NGT to decompress and abx
-delays in diagnosis can cause intestinal ischemia  short bowel syndrome, life long TPN
-operation is a Ladd’s procedure
-rotating counterclockwise
-Ladd’s bands in RUQ to ascending colon is divided
-reposition small bowel on right side and colon on left
-cecum and appendix are in LUQ so usually with appendectomy at same time to avoid misdiagnosis if future appendicitis

Trouble
-delay in diagnosis causing worsening ischemia
-missing duodenal atresia/stenosis intraoperatively, should make sure by sticking NGT through second part of duodenum during surgery
Chapter 33: Infant with nonbilious emesis  pyloric stenosis
Differential

H&P
-projectile, nonbilious vomiting
-usually males at 6 weeks
-sunken fontanelle, palpable mass in epigastrium or RUQ

Pathophys
-unknown etiology
-hypertrophy and hyperplasia of pylorus

Workup
-if no palpable “olive”, get US
-can get UGI contrast study if US still undiagnostic
-expect hypochloremic, hypokalemic, metabolic alkalosis

Management
-pyloric stenosis is a medical emergency not a surgical emergency
-IV access and fluids
-give KCl once child urinates
-surgery delayed until fluid resuscitated and lytes replaced then  Ramstedt pyloromyotomy usually within 24 hours
-can resume oral feeding few hours after surgery
-will vomit some after surgery due to edema in area
-if incomplete pyloromyotomy will have persistent vomiting beyond POD3-4
Trouble
-inadequate resuscitation before surgery since anesthesia can have catastrophic reactions in hypovolemic children
-post-op complications: incomplete pyloromyotomy, pyloric leak if cut too deep into mucosa can develop into peritonitis

Page 43 of 77
Chapter 34: Infant with abdominal wall defect  gastroschisis
Differential

H&P
-risk factors for gastroschisis: young mother, Caucasian, low BMI, singleton pregnancy, recent tobacco use
-risk factors for omphalocele: mother <20 or >40, high BMI, SSRI use
-omphalocele is 1 in 4-5,000 and gastroschisis is 1 in 2-10,000
-gastroschisis vs omphalocele

Pathophys
-gastroschisis thought to be from vascular accident in umbilical ring causing local defect in body wall
-omphalocele is thought to be from arrest of lateral-body fold migration and body wall closure
-gastroschisis has more urgent problems: bowel exposure to amniotic fluid causes inflamed bowel, concurrent malformations or vascular
compromise can cause atresias, obstructions, stenosis, necrosis
-omphalocele has membrane covering bowels so less urgent problems but associated with Trisomy 13, 18, and Cantrell

Workup
-postpartum stabilization
-if omphalocele get echo to see if heart defects

Management
-first: airway protection, thermal support, protection of herniated bowels, fluids, IV access
Page 44 of 77
-small omphalocele <2cm can be repaired surgically
-gastroschisis and large omphalocele need silo placement and gradual reduction into peritoneal cavity
-gastroschisis infants usually get ileus from prolonged intestinal inflammation, will need TPN
-if omphalocele ruptures, management is same as gastroschisis

Trouble
-not recognizing emergencies: compromised airway, not inspecting bowel or missing signs of ischemia after delivery
-not properly protecting bowel, needs thermal support, sterile plastic wrap
-missing abdominal compartment syndrome after repair: can present with low urine output, insufficient ventilation, positive fluid balance
since increased intra-abdominal pressure

Chapter 35: Excessive Drooling in Newborn  Esophageal atresia with tracheoesophageal fistula
Differential for excessive drooling/feeding intolerance in newborn

H&P
-CXR with gastric bubble and OGT tube in upper esophagus, distended abdomen
-if oxygen desaturation while feeding  anatomic/functional problem with nasopharynx, oropharynx, esophagus, stomach
-can also present with respiratory distress or pneumonia

Pathophys
-defect in development of longitudinal tracheoesophageal fold that separates primitive foregut into trachea and esophagus
-associated with cardiac (most common), GU, skeletal/vertebral, anorectal, GI, and palatal anomalies, also with VACTERL and
CHARGE syndrome
-types:

-most common is type C, then pure EA without TEF (type A)


-type E (H-form) may be asymptomatic and won’t present until later in life with recurrent pneumonia or respiratory distress with feeding

Workup
-if stable  CXR with NGT/OGT tube
-no further workup necessary
Page 45 of 77
-if history/CXR unclear/equivocal can get contrast esophagram with CXR

Management
-if respiratory compromise  intubate and ventilation
-preoperative management to minimize aspiration with continuous suction of blind upper esophageal pouch and elevate head
-if pneumonia, give abx and G-tube to prevent further reflux
-healthy infants can do within first few days of life
-severe anomalies or respiratory failure 2/2 pneumonia should have ligation of TEF and G-tube for feeding, delay repair until clinically
stable, can delay several weeks
-before surgery: echo, renal US, xrays to find other anomalies
-surgery: divide fistula tract and repair trachea, anastomosis of esophagus
-complications: leak at esophageal anastomosis, usually heal spontaneously but can take back to OR, can cause stricture, can
do balloon dilation later
-all patients have GERD and increased risk of Barrett’s/esophageal cancer

Trouble
-interrupted IVC  IVR drains into right atrium via azygous system, usual surgery for EA/TEF has to divide azygous vein, but if divided
in interrupted IVC can disrupt all venous return from abdominal viscera and lower extremities
-right sided aortic arch: surgery is right thoracotomy, technically difficult if aorta is right sided
-intubation and mechanical ventilation in TEF  GI distension and worsens respiratory distress

PART XI: SKIN

Chapter 36: Recently Changed Skin Lesion  Melanoma


Differential

Page 46 of 77
H&P
-exposure to UV light (UVB), immunosuppression, fair skin/blue eyes are risk factors
-melanoma risk factors also blistering sunburns
-xeroderma pigmentosum (autosomal recessive)  deficient repair of DNA damaged by UV, leads to melanoma, BCC, and SCC at
early age
-ABCDE for melanoma and ugly duckling sign
-chronic skin inflammation is a risk factor for SCC  Marjolin’s ulcer (chronic open burn wounds), chronic venous ulcers, longstanding
skin infections such as hidradenitis suppurativa and HPP, chronic non-healing wounds
-in non-caucasians melanomas appear on nonexposed skin areas such as palms, soles, mucous membranes, nail regions
-men most common site for melanoma is back, and legs for women

Etiology/Pathophys
-dysplastic nevi syndrome (autosomal dominant), risk for melanoma
-melanoma is from melanocytes (neural crest cells) can be from pre-existing nevus or de novo as a new pigmented lesion
-basal cell CA is the most common skin cancer, then SCC, but melanoma has most deaths
-actinic keratosis is most common precancerous skin lesions
-Bowen’s disease is SCC in situ
-BCC is locally destructive but mets are rare, SCC metastasize but much less commonly than melanoma, most common sites for
melanoma mets are other skin, lung, liver, brain, bone (most common mets to small bowel is melanoma)
-melanoma, staged by Breslow depth by depth of invasion
-superficial spreading (most common)  long horizontal growth phase before vertical growth phase
-lentigo maligna  tumor remains at junction, best prognosis
-acral lentiginous  subungual, sole, or palm location, common in ethnic groups of color, not related to UV light exposure
-nodular  worst prognosis due to rapid vertical growth, increased met potential, 5% amelanotic

Workup
-punch biopsy to get through dermis
-excisional if small
-incisional biopsy for large lesions
-if melanoma  CXR, CBC, LFTs, LDH (prognostic indicator, sign of liver mets)
-if with palpable LAD, CT of chest, abd/pelvis, and PET scan to rule out mets
-MRI brain if neuro signs

Management
-treatment for SCC and BCC: electrodissection/curettage, topical therapies, surgical excision, radiation, cryotherapy
-primary treatment is excisional biopsy
-if cosmetically sensitive area, aggressive features, ill-defined lesions, or recurrent BCC/SCC can do Mohs surgery
-surgical margins for BCC is 3-5mm, SCC is 5-10mm
-if positive margins have to re-excise
-melanoma dx after punch or excisional biopsy need to re-excise for wider margins, SLNB if >1mm thickness
-lymph node dissection if clinically palpable disease or with positive SLNB
-melanoma of fingernail then amputation through joint
-melanoma f/u every 3-6months over first 3 years

Trouble
-missing discolored nail bed as melanoma
-doing shave biopsy when melanoma is suspected
-relying solely on ABCDE for melanoma

Page 47 of 77
Chapter 37: Right leg pain, swelling, erythema for 2 days  necrotizing soft tissue infection
Differential

H&P
-crepitus indicates gas in tissues
-risk factors for necrotizing skin infection are DM, malnutrition, IV drug use, obesity, chronic alcohol abuse, CLL, chronic steroid use,
renal failure, peripheral arterial disease, cirrhosis
-bullae indicate partial tissue death within layers of skin so fluid forms between tissue layers
-violaceous skin implies discoloration 2/2 ischemia
-hard signs for necrotizing skin infection = hypotension, crepitus, skin necrosis/bullae, gas on Xray

Pathophys
-necrotizing soft tissue infection (NSTI) ranges from necrotizing cellulitis, necrotizing fasciitis, necrotizing myositis (also called gas
gangrene)
-type 1 NSTI is polymicrobial infection, type 2 is infection with group A strep, type 3 is clostridial myonecrosis (clostridium perfringens)
-NSTI that involves scrotum or perineum = Fournier’s gangrene

Management
-IV fluids, abx
-surgical debridement
-if extensive muscle necrosis, amputation
-intraoperative findings that confirm NSTI = murky fluid, gray discoloration of fascia, lack of bleeding from fascia, or fascia separating
from muscle too easily
-second look operation within 24 hours to ensure infection has not re-emerged/spread
-if suspicion is high but diagnosis of NSTI is uncertain then surgical exploration is chosen, make sure to incise down to fascia and
muscle to look
-plain xrays to look for gas, CT can also be used
-hyperbaric oxygen is controversial

Page 48 of 77
PART XII: SURGICAL COMPLICATIONS

Chapter 38: Postop bleeding  vWF disease


Differential

Differential for isolated prolonged PTT

H&P
-hx of bleeding after minor trauma/procedure in mouth, epistaxis, bleeding into muscles/joints, menstrual bleeding, after tooth
extractions
-vit K deficiency from short bowel syndrome, cystic fibrosis
-cardiac diseases have antiplatelet meds prescribed

Pathophys
-disorders of primary hemostasis are abnormalities in platelets
-disorders of secondary hemostasis are factor abnormalities
-vWF disease
-type 1 (autosomal dominant)  quantitative deficiency, most common, most mild
-type 2 (recessive or dominant)  quantitative and qualitative, many subtypes, moderate symptoms
-type 3 (recessive)  quantitative, rare, most severe
-exposed/injured subendothelial collagen binds to vWF, platelets bind to vWF using GPIb receptor to make platelet plug for primary
hemostasis
-renal failure, uremic toxins cause platelet dysfunction, can be managed by desmopressin or dialysis
-liver failure, synthetic function of factors, and thrombocytopenia is from sequestration in spleen, deficiency of thrombopoietin, and
immune-mediated destruction of platelets
-vit K factors are II, VII, IX, X, C, S
-DIC from amniotic fluid, sepsis, Auer rods in AML, mucin in adenoCA
-causes of thrombocytopenia

Page 49 of 77
-INR  factors I, II, V, VII, X (common and extrinsic pathways), monitors warfarin
-PTT  factors I, II, V, VIII, IX, X, XI, XII (common and intrinsic), heparin
-anticoagulation medications

Workup
-history, INR/PTT and platelet count

Management
-ABCs, IV access, type and cross, CBC, INR/PTT
-if renal failure, desmopressin initially, then dialysis
-if liver disease, FFP, cryo, coag factors, platelet transfusion
-platelet transfusion when <50,000 and undergoing invasive surgery
-platelet transfusion when <10,000 and asymptomatic
-reverse warfarin  FFP
-stop aspirin 4 days before surgery, clopidogrel 7-10days, warfarin 4-6 days

Complications
-transfusion reactions

Page 50 of 77
-leading cause of transfusion-related fatalities is transfusion-related acute lung injury (TRALI)  non-cardiogenic pulmonary edema,
thought to be from donor Ab attacking recipient’s WBC, the Ab-WBC complex aggregates in vasculature of lungs and causes
inflammatory mediators which increase permeability of lung capillaries causing pulmonary edema
-give fluids, vasopressors
-respiratory support

Chapter 39: Postop decreased urine output  prerenal AKI from hypovolemia
Differential

H&P
-look for op and anesthetic records for events, blood loss, blood products, fluids etc
-azotemia = rise in BUN and creatinine
-normal urine output for adults is 0.5-1mL/kg/hour, oliguria is less than that, anuria is <50-100mL urine/24hours
-most common nephrotoxin meds include IV contrast, aminoglycosides (gentamicin), amphotericin, cisplatin, cyclosporine, NSAIDs

Pathophys
-patients with renal disease and given IV contrast risk for AKI
-can prevent with IV NS prior to contrast (can also do bicarb, N-acetylcysteine)
-oliguria is common after major surgery because stress of surgery causes aldosterone and ADH release causing water/salt retention, but
shouldn’t last more than 24 hours
-most general anesthetics cause myocardial depression and systemic vasodilation, can lead to decrease in cardiac output and end-organ
perfusion

Workup
-BUN and creatinine, ratio >20:1 suggests prerenal AKI
-can get FENa, decreased if prerenal
-high specific gravity suggests volume depletion, red cell casts glomerular disease, hematuria renal emboli/stones, white blood cell casts
infection/inflammation, and granular casts ATN
-can get renal US to assess for post-renal
Management
-rule out obstructed urinary catheter first
-review all medications, nephrotoxic drugs
-fluid challenge: bolus of 500mL-1L NS over 30 min with foley and record output
-prerenal AKI should response with increased urine
-if repeat fluid challenges shows no improvement, assess for postrenal or intrinsic renal etiologies
-if renal failure persists and patient still anuric, dialysis

Page 51 of 77
-emergent dialysis (AEIOU): acidosis, electrolytes (hyperkalemia), intoxication (ethylene glycol), overload (fluid), uremia
-diuretics only beneficial if cardiogenic oliguria (decompensated CHF), otherwise no diuretics
-dopamine not recommended for oliguria
-supposed to increase cardiac output and cause renal vasodilation in normal patient
-in oliguric patients with AKI, can induce renovascular constriction
-in patients receiving diuretics, can potentiate diuresis and cause volume depletion

Trouble
-renal failure that undergo gadolinium-enhanced MRI, risk for nephrogenic systemic fibrosis  marked thickenin and hardening of
skin, fibrosis of internal structures such as muscle, fascia, lungs, heart

Chapter 40: SOB POD5  DVT/PE


Differential

H&P
-Virchow’s triad: stasis, endothelial injury, hypercoagulable stage
-Well’s score >4 points indicates likely PE

-Homan’s sign very low sensitivity


-left leg 2x more likely for DVT because left iliac vein is often compressed by right iliac artery  May Thurner syndrome
-causes of postop fever

Pathophys
-commonly inherited hypercoagulability: factor V Leiden, prothrombin mutation are most common
-also protein C or S deficiency, antithrombin deficiency, homocysteinemia

Page 52 of 77
-protein C or S deficiency risk of warfarin-induced skin necrosis because warfarin reduces C, S first causing transient
prothrombotic state causing skin necrosis
-acquired causes of hypercoagulability: age, pregnancy, malignancy, OCPs, HRT, smoking, obesity, nephrotic syndrome, HIT (acute
drop in platelets after heparin or LMWH within past 4-10days
-pulmonary edema
-cardiac etiology: usually associated with acute cardiac event such as MI, LV failure, dysrhythmia, can hear S3, JVD, crackles,
cool/pale extremities, definitively dx by PCWP >18mmHg
-noncardiac etiology: extremities warm, perfused, PCWP nonelevated, treat with ventilator support
-wide A-a gradient: atelectasis, pneumonia, PE

Workup
-Well’s score
-heparin  will not break clot but will prevent it from progressing
-CT angiogram
-if suspicion for VTE is low then get D-dimer assay to see breakdown of fibrin
-high negative predictive value, if negative can rule out PE
-if positive, order CT angio
-if CT angio nondiagnostic then can get Doppler for legs, with/without V/Q scan
-if critically ill and can’t transport can get echo to show right heart strain, or can get Doppler of legs
-pulmonary embolism findings
-ABG: respiratory alkalosis, hypoxemia, increased A-a gradient
-ECG: sinus tachy, rarely will see large S wave in lead I, large Q wave and inverted T wave in lead III
-CXR: normal, rarely will see Westermark’s sign (pulmonary oligemia distal to embolus), Hampton’s hump (wedge-shaped
density at periphery of lung)
Management
-LMWH or heparin
-if history of HIT, give lepirudin, argatroban, bivalirudin (direct thrombin inhibitors)
-if contraindication to anticoagulation  IVC filter
-treatment can be heparin, tPA, endovascular clot aspiration, or open pulmonary embolectomy
-tPA for massive and submassive PE, not for low-risk PE
-contraindications include surgery within last 2 weeks, ICH or malignancy, ischemic stroke within last 3 months,
suspected aortic dissection, active bleeding, significant closed head trauma within 3 months, severe HTN >185
-open pulmonary embolectomy if massive PE and not tPA candidates
-heparin/LMWH 5 days after PE, warfarin started in 1 st/2nd day to overlap for 4-5 days, goal INR 2-3, anticoagulation for at least 3
months
Trouble
-if patients with malignancy, LMWH is better than warfarin
-pregnancy use LMWH/heparin since warfarin is teratogenic
PART XIII: TRAUMA

Chapter 41: Abdominal pain after MVC  hypovolemic shock


Shock in a Trauma Setting

H&P
-signs of shock: tachycardia (initial), hypotension, pale/cool extremities, weak pulses, prolonged capillary refill (>2s), low urine output,
AMS

Page 53 of 77
-blood at urethral meatus after blunt trauma usually means urethral injury 2/2 pelvic fracture, can also have perineal ecchymosis, scrotal
hematoma, high riding prostate on DRE
-foley is contraindicated, should do retrograde urethrogram to evaluate
-gross hematuria after blunt trauma  injury to kidney or bladder
-r/o with CTAP with contrast, bladder evaluated by CT cystogram

Pathophys
-Shock class I-IV

-5 major sources of blood loss in blunt trauma  chest, abdomen, pelvis/retroperitoneum, long bones, and “street” or external
-chest: hemothorax from lung laceration or torn intercostal arteries
-abdominal: liver trauma, spleen rupture
-retroperitoneum: pelvic fractures that tear arteries off internal iliac arteries or pelvic veins, renal trauma, blunt trauma to
abdominal aorta/IVC is rare
-long bones: femur fracture
-also large scalp lacerations can lead to a lot of blood loss
-closed-head injury should NOT be considered the source of hypovolemic shock
-would cause Cushing reflex (HTN and brady) via sympathetic response causing peripheral vasoconstriction to
maintain BP to head, because of vasoconstriction and HTN, the baroreceptors respond by causing bradycardia  seen
in increased ICP and heralds brain herniation

Management
-primary survey: ABCDE
-Airway with C-spine precaution
-Breathing: look for symmetrical chest movement, injury, auscultate bilaterally, and palpate for crepitus or chest deformity
-Circulation: if radial pulses SBP >80, carotid/femoral pulses SBP >60, IV access (two 16 gauge) try for 1 in each antecubital
fossa if able
-central line only if peripheral access is problematic and patient is not HDS, want femoral vein for access
-Disability: neuro evaluation, GCS (eye, verbal, motor)
-Exposure/environment: remove clothing for complete primary survey
-secondary survey: if patient is conscious and speaking, AMPLE (allergies, medications, past medical hx, last meal, events preceding
trauma), careful and systematic head-to-toe exam
-airway: rapid sequence intubation (RSI) and C-spine protection
-weight-based sedatives  etomidate and neuromuscular blocking agents without bagging
-surgical airways: cricothyrotomy and tracheostomy
-cricothyrotomy is the choice in EM setting, easier/faster, less complications
-tracheostomy is the choice for more long-term management
-proper intubation confirmed by end-tidal CO2 (capnography) but is not accurate if cardiac arrest
-CXR to confirm not past carina
-if patient doesn’t respond to initial fluid resuscitation or is transient, means still actively bleeding  start giving Oneg
-after type/cross can give 1:1 or 2:1 with FFP
-should go to OR, if awake/alert with symptoms can do ex lap
-if unresponsive should do FAST scan, if positive to OR for ex lap
-CT scan is contraindicated in an unstable patient
-diagnostic peritoneal lavage if FAST is equivocal or no FAST scan available
-small incision in abdomen then catheter, aspirate first if >20cc blood then go to OR
-if no blood on aspiration, can lavage with 1L NS, if >100,000 RBC/mL on return then positive

Page 54 of 77
-cannot detect retroperitoneal bleeds
-most common intra-abdominal bleed after blunt trauma is splenic injury
-if HDS can do splenic embolization
-if not HDS, ex lap and splenectomy
-vaccinate for Strep pneumo, H flu, Neisseria meningitidis 2 weeks postop
-most common injury from blunt trauma is liver
-if HDS can do embolization
-if not HDS, surgical exploration with perihepatic packing, cauterization, suturing
-pelvic fracture  pelvic binder/sheet around greater trochanter of femurs, angiography with embolization if everything else stable

Trouble
-free fluid in peritoneal cavity with no solid organ injury  occult source (SMA), enteric contents (bowel injury), or urine (bladder
rupture)
-vasopressors is NOT recommended in hemorrhagic shock
-hypotension cut-offs
-<90-100 if age 20-49yo
-<120 if age 50-69
-<140 if >70

Chapter 42: penetrating abdominal trauma  hemorrhagic shock, peritonitis 2/2 bowel injury, left iliac artery injury
H&P
-prehospital report MIVT: mechanism, injuries, vitals, treatment
-two most common penetrating traumas: stab wounds and GSW
-stab wound damage is isolated to pathway of penetration, localized injury
-smaller hand guns deliver low velocity injuries, high power shotguns can cause high velocity injuries causing tissue damage
in areas remote from bullet track
-immediate surgery with penetrating abdominal trauma and hypotension, peritonitis, or evisceration
-tangential GSW = identifiable entry and exit wounds without clinical evidence of injury to deeper structures, not all need to go to
surgery

Anatomy
-3 regions of internal abdomen: peritoneal cavity, pelvis, retroperitoneum
-retroperitoneal organs, injuries often missed by FAST
-duodenum 2nd-4th parts
-pancreas
-kidneys/ureters/bladder
-ascending colon and descending colon
-rectum, distally

-most common organ injuries from penetrating abdominal injury is small bowel then liver
-GSW that travels across pelvis needs to do proctoscopy to r/o injury to rectum, vaginal exam for females, then CTAP

Workup
-penetrating abdominal trauma with hypotension, peritonitis, or evisceration  OR
-if penetrating abdominal trauma without above, CTAP with contrast, CT chest if torso trauma
-FAST usually for blunt trauma, DPL more for blunt abdominal injury
-FAST not good for retroperitoneal injuries
-if injury has not penetrated anterior fascia, surgery can be avoided Management

Page 55 of 77
-ABCDE
-MTP = massive transfusion protocol with PRBCs, plasma, platelets
-with penetrating torso trauma and hypotension, might limit fluids to avoid “popping the clot”  permissible hypotension
-abx and analgesics only if clear indicates for OR otherwise might mask signs
-tetanus if patient <3 doses of tetanus toxoid or status unknown with tetanus-prone wound
-impalement should be assessed same as penetrating, don’t remove until in OR
-in OR draped from chin to knees, need to leave access to left chest in case, and to groin for access if needed
-ex lap is traditional mainstay for penetrating abdominal injury
-can do laparoscopy if without hypotension, peritonitis, evisceration to evaluate
-non-op management for penetrating abdominal trauma
-HDS
-no peritonitis
-normal mental status
-CT showing no intra-abdominal injury
-triad of death: acidosis, hypothermia, coagulopathy
-damage control surgery = might need to stop surgery with no fixes, bring to ICU to correct the lethal triad before going back
to OR
-if arterial injury with stool contamination, use vein graft (reverse saphenous vein)

Complications
-abdominal compartment syndrome = severely injured patient with decreased urine output, increasing peak pressures on ventilator,
increasing vasopressor support in the absence of other causes, bladder pressure can be measured and reflects intra-abdominal pressure
-treat by going to OR  decompressive laparotomy, open abdominal fascia, leave wound open

Chapter 43: pedestrian struck by motor vehicle, right knee deformity  popliteal artery injury
H&P
-hard and soft signs for vascular injury

-6 P’s of limb ischemia  pain, pallor, paresthesia, paralysis, pulselessness, poikilothermia (only pulselessness is a hard sign for vascular
injury)
-audible bruit/palpable thrill near artery with trauma  traumatic AV fistula

Pathophys
-popliteal artery is tethered at the superior border (superficial femoral artery exits adductor hiatus) and inferior border (tendinous arch
of soleus), and forms anastomotic network around knee
-posterior dislocation of knee compresses popliteal artery against posterior aspect of tibial plateau
-clinically silent injuries = discovered on radiographic studies (angiography or US) but no clinical signs
-common skeletal and associated artery injuries

Page 56 of 77
Workup
-if presence of hard signs of vascular injury no imaging studies are needed  OR for hemorrhage control
-if absence of hard signs then ankle:brachial index = Doppler of SBP of lower vs upper extremities
-normal ABI = 1-1.2
-ABI <0.90 is suggestive of arterial injury
-if ABI <0.9, need more vascular imaging
-formal contrast arteriography (IR), CT angiography (first choice), duplex US
-xray of knee after reduction
-mangled extremity severity score (MESS) = age, severity of shock, mechanism of injury, skeletal/soft tissue injury, ischemia severity
-assess for how salvageable limb is

Management
-ABCDE
-reduction of knee, complete exam after reduction, neurovascular status after exam
-if no pulse after reduction, start IV heparin (unless actively bleeding/other contraindications)
-systemic heparin because want to prevent microvascular thrombosis in setting of low-flow arterial circulation
-check ABI after heparin
-if ABI still <0.9, get CTA
-if CTA says arterial injury  OR (if need graft, greater saphenous vein)
-if both orthopedic and vascular injury, can insert intravascular shunt before orthopedic stabilization for temporary blood flow
-compartment syndrome is a complication
-serial Doppler US for arterial occlusion
-prophylactic fasciotomies

Chapter 44: GSW to left neck


H&P
-neck injury want: mechanism of injury, location of injury, clinical exam findings
-hard signs for vascular injury = active arterial bleeding, hematoma, shock, palpable thrill/audible bruit  OR
-findings
-stridor: upper airway obstruction of some sort  immediate attention to airway, intubate
-odynophagia: pain with swallowing suggests injury to oropharynx/esophagus
-Horner’s syndrome: ptosis, miosis, anhydrosis (sympathetic innervation that travels with common carotid/internal carotid
arteries)
-thrill/bruit: damage to subclavian, carotid artery or vein to create AV fistula
-crepitus: subcutaneous emphysema 2/2 injury of aerodigestive tract or lungs
-hoarse voice: dysfunction of vocal cords from direct trauma or damage to vagus or recurrent laryngeal nerve

Page 57 of 77
Anatomy
-zones of neck

-carotid sheath: common carotid artery, IJ vein, vagus nerve

Pathophys
-injuries that don’t penetrate platysma = nonpenetrating neck injuries, no further workup
-artery injuries from bullet: transection, pseudoaneurysm, intimal injury, dissection, AV fistula
-intimal injury: concussive/blast effect of bullet can damage intima of artery, if large can occlude lumen causing thrombosis,
can also create false lumen and create a dissection
-pulsating mass: AV fistula, aneurysm, pseudoaneurysm
-if vocal cord paralysis then injury of recurrent laryngeal nerve (all intrinsic laryngeal muscles except cricothyroid)
-bilateral paralysis of vocal cords can cause complete upper airway obstruction
-phrenic injury can cause elevated diaphragm on affected side

Workup
-penetrating neck trauma imaging
-CT angio
-duplex US
-femoral catheter angio
-triple endoscopy = laryngoscopy, esophagoscopy, bronchoscopy

Management
-ABCDE
-if hard signs (expanding hematoma, hypotension, exsanguination)  OR
-if no hard signs  helical CT angio
-if CT negative, no surgery
-if CT is not definitive then triple scope
-if cannot control arterial bleed in ER, can put foley and inflate balloon then OR
-zone 2 injuries have lower threshold for surgery since location is readily accessible/explorable (zone 1 needs chest incision and zone 3
is at skull base)
-carotid injury: find proximal and distal ends, give systemic heparin (if no contraindications) can try primary anastomosis, or graft
(greater saphenous)
-external carotid artery injury can be ligated
-IJ vein should be repaired but if cannot be, can ligate
-pseudoaneurysm and AV fistula need repair
-intimal injury is usually stable and will heal spontaneously
-if blunt carotid injury
-present with focal neurological deficit not explained by head CT
-CT angio of neck
-usually treatment is anticoagulation only
Page 58 of 77
Chapter 45: stab wound to chest  cardiac tamponade and tension pneumothorax
Lethal 6 Injuries of Thoracic Trauma

Hidden 6 Injuries of Thoracic Trauma

H&P
-combative patient can be intoxicated but also hypoxia, hypovolemia/cardiogenic shock, hypoglycemia can also cause
-penetrating injury above nipple line likely involves only thoracic structures, injury below nipple line may damage thoracic structures,
abdominal contents, diaphragm itself
-pulse pressure <30 is narrow, compromised stroke volume, think about tamponade, hypovolemic or cardiogenic shock
-sucking chest wound means open pneumothorax = chest wall defect is so large (>2/3 diameter of trachea) that inspired air goes through
wound instead of through trachea
-spontaneous pneumo: young, tall, thin, male smokers from spontaneous rupture of apical alveolar blebs
-simple vs tension vs iatrogenic PTX
-subcutaneous emphysema in trauma is from pneumothorax until proven otherwise

Pathophys
-JVD in trauma  look for tension PTX or cardiac tamponade
-penetrating injury below nipple, need to rule out intra-abdominal injuries to colon/bowel etc
-air embolism from penetrating chest trauma can present with CV signs (chest pain, arrhythmia, right sided HF), resp signs (dyspnea,
hypoxemia, hypercarbia), and CNS signs (confusion, AMS)
-can be from concomitant injury to a bronchus and pulmonary vein, fistula between them, also if on high positive pressure
ventilation
-stab wounds or GSW near the hilum are most common mechanism of such an injury due to their close proximity to each other

Management
-ABCDE
-don’t intubate and give positive pressure ventilation if suspected tamponade
-needle decompression in tension PTX
-needle in 2nd or 3rd intercostal space above rib at midclavicular line and advanced until air is aspirated into syringe
-then get chest tube after needle decompression
-tube thoracostomy (chest tube) is between 4th or 5th intercostal space at midaxillary line
-chest tube with >1.5L immediately or >150-200mL/hr for 3 hours = massive hemothorax  OR

Page 59 of 77
-emergency department thoracotomy (EDT) for hemorrhage control, decompression of tamponade, cross-clamping descending thoracic
aorta, cardiac massage, repair of cardiac/pulmonary injuries
-if cardiac tamponade
-IV fluids to increase preload
-OR  median sternotomy, release of tamponade and repair of underlying injury
-EDT if patient loses their vital signs before OR, pericardiocentesis is not recommended in trauma setting
-subxiphoid window if unsure of tamponade (FAST equivocal) to see if blood in pericardium, if present can convert to sternotomy
-subxiphoid window contraindicated in unstable patient
-combative patient order: glucose, pulse ox, and get vital signs
-inspiring 100% O2 to help rapidly resolve a PTX – decreases alveolar partial pressure of nitrogen and washes out nitrogen from tissue
and increases O2 uptake into vascular system, increased gradient between alveolar capillaries and the PTX will cause accelerated rate
of absorption from pleural space
-sucking chest wound  cover defect taped on 3 sides so air doesn’t enter on inspiration but can exit during expiration to re-expande
the lung, should do chest tube after
-flail chest  analgesics, thoracic epidural, intubation with PEEP if oxygenation/ventilation is compromised

Complications
-pericardiocentesis can lacerate a coronary vessel, can worsen tamponade
-can also injure left phrenic nerve, which passes posteriorly to pericardium of LV
-if recurrent/persistent hemothorax with chest tube  VATS
-if leave hemotrhoax lung cannot fully re-expand and can become infected and cause empyema
-if chest tube doesn’t resolve PTX: make sure in right location and there’s no kink/obstruction
-if still not resolving, consider major airway injury such as disruption of bronchial tree, needs intubation, diagnostic
bronchoscopy and repair by thoracotomy

Trouble
-missing right-sided diaphragm injury  liver will prevent abdominal contents from herniating into chest acutely but with time will
have hernia and bowel in chest
-presentation is chest pain and SOB in patient with remote history of trauma, get CXR
-missing intra-abdominal injury with penetrating wound below nipple, should get CXR, CT scan if stable patient

Chapter 46: Burns to face, trunk, extremities


-singed nasal hairs and carbonaceous sputum with low O2 sats are concerning for inhalational injuries  intubate and start 100% O2

H&P
-Levels of burn injury

-rule of 9’s  head 9, each arm 9, each leg 18, anterior torso 18, posterior torso 18
-suspicion for inhalational injury when with facial burns, change in voice quality, singed nasal hairs, carbonaceous sputum
-inhalational injury: upper airway edema, ARDS, CO poisoning
-CO poisoning: headaches, nausea/dizziness, cherry-red skin, can progress to seizures, coma, multiorgan failure, and death
-burn wound sepsis when 2nd degree burn progresses to 3rd degree burn, can also have discolored burn, eschar with green pigment, black
necrotic skin, skin separation, signs of sepsis
-circumferential full-thickness burn on extremity/chest concern for compartment syndrome  escharotomy
-increased mortality in burn patients: children/elderly, >40% non-superficial TBSA, inhalational injury

Page 60 of 77
Pathophys
-causes of burns
-thermal burns
-chemical burns; alkali worse
-alkali penetrates tissues more deeply, liquefactive necrosis
-acidic burns cause coagulation necrosis
-electrical: immediate concern is arrhythmia, can also have muscle necrosis, posterior shoulder dislocations, myoglobinuria,
renal failure
-DC current (lightning) puts patient at risk for asystole
-AC current (wall socket) risk for V-fib
-long term complication is cataracts
-burn in first 24 hours: release of catecholamines, glucose is a little high, cardiac output is decreased, volume down partly from capillary
leak of fluid also injured tissue destructs RBCs
-risk for GI ulcers from decreased volume/perfusion to GI tract causing ischemia  Curling’s ulcer in duodenum
-burn wound infections  Pseudomonas, Staph aureus, Strep pyogenes, and fungal infection in later stages of recovery (Candida), HSV
is most common viral infection

Workup
-inhalational injury diagnosed by bronchoscopy definitely
-CO poisoning diagnosed by CO pulse ox, standard pulse ox cannot differentiate oxygen and CO bound to Hgb
-burn wound infection by punch biopsy and >10^5 bacteria/gram of tissue with clinical signs of infection

Management
-inhalational injury  intubate
-Fluid for burn victim in first 24 hours, give first half during 8 hours from injury and last half in next 16 hours
Total fluid volume = 4mL/kg x weight (kg) x TBSA(%)
-use LR for burn victims (NS will cause acidosis in burn victims)
-titrate to urine output 0.5mL/kg/hour in adults and 2-4mL/kg/hour in kids
-CO poisoning give 100% oxygen on non-rebreather face mask
-need to monitor Na and K in burn patients; Na because can have compartmental fluid shifts and hyponatremia can cause seizures, K
from destruction of cells/tissues
-circumferential chest burn with deteriorating respiratory status  escharotomy
-don’t need prophylactic IV abx
-topical agents for burn patients
-silver sulfadiazine (silvadene)  leukopenia and thrombocytopenia, ineffective for Pseudomonas, poor deep tissue pentration
-sulfamylon or mafenide acetate  carbonic anhydrase inhibitor can cause metabolic acidosis, good for Pseudomonas, good
deep tissue penetration
-silver nitrate  brown staining of skin, can have methemoglobinemia poor deep skin penetration, ineffective for Pseudomonas
-start PPI or H2 blockers on all burn patients to prevent Curling’s ulcers
-all burn patients should start tube feedings
-chemical burns: remove all clothes, irrigate well
-electrical burns: cardiac monitoring for 12-24 hours

Trouble
-chronic nonhealing wound are at increased risk for SCC of skin (Marjolin’s), skin biopsy to evaluate

Chapter 47: Severe right leg pain after tibia fracture  compartment syndrome
Differential

Page 61 of 77
H&P
-compartment syndrome P’s  pain, paresthesia, pallor, pulselessness, poikilothermia, paralysis
-first sign is usually pain, then nerve ischemia (sensory before motor), pulselessness is a late sign

Pathophys
-compartment syndrome begins with extremity injury, inflammatory mediators result in interstitial accumulation of serous fluid or
bleeding into the compartment, as pressures exceed 5-10mmHg then venules collapse and venous HTN results, perfusion stops, ischemia
begins, can lead to tissue necrosis and permanent limb damage
-tissue damage can cause hyperkalemia, acidosis, myoglobinuria and kidney failure
-lower leg injuries the anterior compartment is most susceptible to compartment syndrome, deep peroneal nerve is in anterior
compartment and supplies motor fibers and sensory to first web space of toes
-Volkmann’s ischemic contracture = sequela of untreated compartment syndrome usually in children after supracondylar fracture,
ischemic muscles become fibrosed and contracted causing a claw-like hand with damaged nerves
-abdominal compartment syndrome can present with impaired respiration from pressure, can also compress IVC and cause decreased
cardiac output and stroke volme

Workup
-diagnosis is clinical
-abdominal compartment syndrome is harder to find clinically, measure bladder pressure, if >25-30mmHg is suggestive

Management
-Extremity – decompressive fasciotomy for all compartments of affected limb
-deep posterior compartment most often missed in lower leg
-abdominal – decompressive laparotomy

Complications
-lower leg fasciotomy
-wound infection
-superficial peroneal injury in lateral compartment  foot drop
-incomplete fasciotomy

Page 62 of 77
PART XIV: UPPER GI

Chapter 48: bloody emesis ulcer


Differential for UGIB

H&P
-bright red bloody emesis, coffee-ground emesis
-worse prognosis in elderly, other comorbid conditions, shock on presentation, large ulcer >2cm

Pathophys
-esophageal varices from portal HTN
-normal distal veins in submucosa empties into left gastric vein (also coronary vein) that drains into portal vein
-if cirrhosis then portal HTN, blood goes in retrograde direction
-acute gastritis from inflammation of stomach lining from impaired mucosal defenses
-chronic gastritis
-type A is fundus-dominant, associated with pernicious anemia
-type B is antral-dominant, caused by H. pylori
-Dieulafoy’s lesion is vascular malformation in which a large tortuous artery is aberrantly located in submucosa on lesser curvature of
stomach is eroded by gastric acid, identified endoscopically
-peptic ulcer penetration
-gastric ulcer through posterior stomach  splenic artery
-gastric ulcer through lesser curvature  left gastric artery
-duodenal ulcer through posterior wall of D1  gastroduodenal artery
-considered upper GI bleed until ligament of Treitz (start of jejunum)

Workup
-Hgb/Hct, can be normal since losing all proportionately
-BUN/crt ratio increases

Management
-IV access and NGT, lavage
-type and cross
-EGD within 12 hours of admission
-if can’t see on EGD consider obscure bleed, can do angiography with embolization for faster bleed, capsule endoscopy, or tagged RBC
if small and lower GI bleed
-if EGD fails after 2 attempts  surgery

Page 63 of 77
-surgical options for bleeding ulcer
-gastric ulcers – excise part of stomach
-duodenal – ligate GDA, truncal vagotomy and pyloroplasty if stable patient to reduce stimulation of HCl production
-all ulcers test for H. pylori
-triple therapy = PPI, clarithromycin, amoxicillin for 1 week
-confirm eradication in 4-6 weeks with urea breath test (not on PPI, can give false negative)
-variceal bleed:
-short term abx
-endoscopic band ligation, repeat EGD in 48hours to band any remaining vessels
-can give somatostatin or vasopressin
-TIPS if failed medical/endoscopy therapy
-long-term give beta blocker to prevent recurrent bleeds
-Mallory-Weiss tear from vomiting/retching/straining/coughing, bleeds are almost always self-limited

Trouble
-assuming esophageal varices in alcoholic  can also be Mallory-weiss tear
-failing to recognize severity of liver disease  coagulopathy

Chapter 49: Severe epigastric abdominal pain perforated ulcer


Differential

H&P
-gastric ulcers have pain during food consumption, duodenal ulcers feel better with eating
-perforated: acute onset of sharp abdominal pain within epigastric region that becomes diffuse, shoulder pain from diaphragm irritation,
peritonitis

Pathophys
-from H. pylori, NSAIDs
-smoking, alcohol are risk factors
-Cushing and Curling stress ulcers from head and burn patients, respectively
-types of gastric ulcers: type 1 is on lesser curvature (most common), type II body of stomach or duodenum, type III prepyloric antrum,
type IV within cardia near GE junction and tend to be highly painful, type V associated with NSAIDs and can be anywhere in stomach
-type 2 and 3 have acid hypersecretion

Workup
-clinical findings, CXR for free air
-CBC, BMP, BUN/crt, amylase and lipase, LFTs
-CT with contrast can confirm

Management
-IV abx, PPI, fluids
-omental patch
-if perforated but <12hour since onset of symptoms, HDS, <70yo, no history of failed medical therapy, no associated comorbidities, and
radiologic documentation of sealed perforation can manage non-operatively  NGT, abx, PPI, observation

Page 64 of 77
Chapter 50: Weight loss and early satiety  gastric cancer
Differential

H&P
-gastric cancer rate is higher in Eastern Asia (Japan, Korea, China), male predominance, gastric adenocarcinoma is 90% of all gastric
cancer
-risk factors fam hx, diet (nitrates, salt, fat), familial polyposis, gastric adenomas, HNPCC, H. pylori, smoking, menetrier’s disease, type
A blood, BRCA1 and 2, HER2 gene, Petuz-Jegher’s syndrome
-weight loss and abdominal pain are the most common symptoms at initial diagnosis

Pathophys
-types histological types of gastric adenoCA
-intestinal type gastric adenocarcinoma: from gastric mucosa, usually in distal stomach, usually in sporadic patient with poor
diet, smoking, alcohol, other risk factors, one associated with H. pylori, develops from histologic progression from
inflammation to chronic gastritis to metaplasia/dysplasia then carcinoma
-diffuse type gastric adenocarcinoma: poorly differentiated tumor from lamina propia that grows in infiltrative pattern,
thickening without discrete mass, more associated with congenital disorders, not a histological progression
-highly metastatic and aggressive can cause  linitis plastica
-vascular supply to stomach is left and right gastric arteries on lesser curve and left and right gastroepiploic arteries along greater curve,
all derived from celiac axis
-gastrointestinal stromal tumors (GIST): mesenchymal tumors of variable malignant potential, usually smooth submucosal masses, near
universal expression of c-KIT and CD117
-gastric carcinoid: neuroendocrine tumors, usually not in stomach, if functional can cause gastrinoma/Zollinger-Ellison syndrome which
needs to be resected
-gastric lymphoma: most common is MALT and diffuse large B cell lymphoma
-MALT usually with H. pylori, give triple therapy
-diffuse large B cell  CHOP (cyclophosphamide, doxorubicin, vincristine, prednisone, radiation)

Workup
-EGD with biopsy
-no need to biopsy if thinking GIST
-if biopsy(+)  EUS for lymph nodes and regional disease, CT scan of abdomen for mets and surgical candidate/anatomy
-can do PET scan for mets/lymph node

Management
-resection, 5cm margins
-wedge resection for GIST, 2cm margins
Page 65 of 77
-remove at least 15 lymph nodes for staging
-gastroadenocarcinoma: HER2 testing  trastuzumab
-GIST: give tyrosine kinase inhibitor (imatinib for c-KIT)

Complications
-dumping syndrome after gastric resection  caused by rapid distribution of food within small intestine in the absence of pyloric
sphincter regulation, hyperosmolar state causes increased water secretion causing diarrhea and eventually hypotension
-other complications: early satiety, anastomotic leak, afferent limb syndrome, internal hernia, SBO
-anastomotic leaks will present with abdominal pain, peritonitis, fevers, tachy, leukocytosis, sepsis if not treated, order upper
GI with contrast, contrast extravasation will confirm leak, if leak is present  re-operation, keep patient NPO and on TPN after
operation

Chapter 51: Chest pain after vomiting  Boerhaave’s


Differential

H&P
-risk factor is alcoholics or over eating with aggressive vomiting, usually in 50-70yo males
-Mackler’s triad = vomiting, thoracic pain, subcutaneous emphysema
-subcutaneous emphysema after retching is pathognomonic for esophageal rupture
-Mallory-Weiss is a partial tear of mucosa at GE junction but Boerhaave’s is full-thickness rupture
-mallory-weiss presents with UGI bleed, Boerhaave’s presents with pain
-Mallory-weiss resolves spontaneously, rarely need surgery, Boerhaave’s require immediate surgical repair

Pathophys
-majority of esophageal perforations are iatrogenic from EGD, or others can be from penetrating trauma, foreign body ingestion,
malignancy with perforation
-sepsis is from gross contamination of mediastinum, can also get to pleura

Workup
-get CXR
-most common findings are left-sided pleural effusion and atelectasis
-to confirm can get CT with oral contrast (water-soluble contrast not barium!)
-no EGD!!!

Management
-NPO, IV fluids, broad spectrum abx
-H2 blocker or PPI
-ideally treatment within 24hours
-transthoracic/transhiatal repair, pleural decortication with flap of pleura or intercostal muscle, reconstruct esophagus can be
from colon or jejunum
-if >24 hours, closure over T-tube or exclusion+diversion
Page 66 of 77
-nonoperative management with NGT on suction, IV abx, and TPN for those with minimal comorbidities, no signs of sepsis, and
perforation <24hours, and contained leak
-can re-image on day 7 to see closure of perforation
-continue PO abx for 6-8weeks

PART XV: UROLOGY

Chapter 52: Scrotal pain  testicular torsion


Differential

H&P
-blunt trauma to testicles is most common
-cremasteric reflex is elevation of ipsilateral testicle by cremasteric muscle in response to stroking motion on medial aspect of thigh, if
provoked sensory fibers from femoral branch of genitofemoral nerve (L1-L2) are stimulated, synapses with motor nerve from genital
branch of genitofemoral nerve (L1-L2) to activate cremasteric muscles and elevates ipsilateral testis
-cremasteric reflex is absent with UMN and LMN disorders, with spinal cord injury at L1-L2, and in patients with testicular
torsion
-Prehn’s sign when patients report relief of pain with elevation of scrotal contents, negative sign when no relief of pain
-if no relief of pain (negative Prehn’s) then more testicular torsion
-if positive then more likely epididymitis
-blue-dot sign is pathognomonic for torsion of testicles or epididymal appendage, palpation of testis reveals small firm and tender nodule
near head of epididymis that appears to have blue discoloration
-4 cardinal signs for testicular torsion  n/v, testicular pain <24 hours, superiorly displaced testicle, absent cremasteric reflex
-appendix testes torsion = epididymal appendate is located at head of epididymis, presents with gradual onset of pain, usually acute
painful hemiscrotum of child, blue-dot sign is classic finding, cremasteric reflex is present, can be treated with NSAIDs and ice packs

Pathophys
-defects in processus vaginalis can lead to failure of testes to attach to scrotum
-patients with bell-clapper deformity = failure of normal posterior anchoring of gubernaculum, testes, epididymis, allowing testes to
freely rotate and swing within tunica vaginalis, usually present in both testes

Workup
-clinical diagnosis
-if suspicion is low
-get UA to r/o UTI

Page 67 of 77
-Doppler US of scrotum is imaging of choice  should show absence of arterial blood flow in affected testis
-in trauma look to see if tunica albuginea is violated (testicular rupture)

Management
-immediate urology consult
-best within 4-6 hours, 20% viability after 12 hours
-if symptoms <6hours, manual detorsion in ER (twist laterally) then elective orchiopexy
-if significant delay until diagnosis should go to OR for reduction of torsion, untwist, and orchiopexy
-if necrotic in OR  orchiectomy
-if minor trauma to testes can treat with scrotal support, ice packs, NSAIDs, rest
-testicular trauma, OR if
-suspicion for violation of tunica albuginea
-expanding testicular hematoma
-avulsion, scrotal degloving, absence of blood flow on Doppler US
-surgical exploration if penetrating testicular trauma
-loss of 1 testicle does not affect fertility

Trouble
-failing to do a scrotal exam in adolescent male who presents with abdominal pain and vomiting

Chapter 53: Scrotal mass  testicular cancer


Differential for scrotal masses of skin

Differential for scrotal masses of spermatic cord

Differential for scrotal mass of epididymis

Differential for scrotal mass of testes

Page 68 of 77
H&P
-testicular cancer usually has painless mass, most are young adults (20-35yo), mass is within testicle not separate from it
-rarely will have gynecomastia 2/2 to hormonally active tumor
-if constitutional symptoms then most likely metastatic disease
-not really any risk factors, except for fam hx, Klinefelter’s
-extremely painful scrotal mass should think epididymitis and/or orchitis
-spermatocele, varicocele, and hydrocele are painless
-varicocele will disappear when lying down and reappear when standing up, feels like spongy bag of worms, associated with
infertility

Pathophys
-risk of testicular cancer in undescended testicle and also risk in contralateral descended testis
-seminoma (germ cell tumor) is most common subtype and is malignant

-epididymitis bugs – chlamydia trachomatis (serotypes D-K), Neisseria gonorrhea (E. coli, pseudomonas if older adult)
-choriocarcinoma can cause ectopic hCG which can stimulate breast development/gynecomastia
-alpha subunit of hCG similar to TSH so can also have symptoms of hyperthyroidism
-epididymitis is an acute process can be confused with torsion
-spermatoceles development from retention cyst, will transilluminate with light
-varicocele is from impaired venous drainage
-pampiniform plexus veins progressively dilate and enlarge, more often on left since left testicular vein goes into left renal vein
at a right angle, increases testicular temperature and damages sperm
-sudden left-sided varicocele think thrombosis of left renal vein (left renal cell carcinoma)
-hydrocele – fluid can accumulate in tunica vaginalis or persistent processus vaginalis
-if communicate with peritoneum will change in size with Valsalva
-common in newborns and resolves spontaneously within first year of life as tunica vaginalis separates from processus vaginalis

Workup
-testicular US  if solid mass within testicle most likely cancer, while purely cystic fluid-filled mass is unlikely malignant
-stage with CT scan of abd/pelv and CXR, if CNS symptoms also CT/MRI of brain
-order beta-hCG, AFP, LDH
-AFP elevated in nonseminomatous cancer
-no biopsy, might seed

Management
-radical inguinal orchiectomy
-radiation, chemo, retroperitoneal lymph node dissection
-seminomas highly radiosensitive  radiation
-retroperitoneal lymph node dissection primary for nonseminomas
-both respond to chemo

Page 69 of 77
-chemo patients can have infertility, can do cryopreservation of sperm
-unilateral orchiectomy does not affect erectile function

Chapter 54: Blood in urine  urologic malignancy


Differential for gross hematuria

H&P
-bright red, thick consistency of urine  moderate/severe active bleeding
-pink  mild active bleeding
-brown  old blood; glomerular bleeding
-beets, rhubarb, drugs (rifampin, sulfonamides, nitrofurantoin, phenytoin, metronidazole, others) can make urine red, also rhabdo can
give urine red appearance
-dark urine can be seen with elevated levels of conjugated bilirubin
-pain with hematuria most likely infection or urinary obstruction/stone

Page 70 of 77
-risk factors for kidney stones is fam hx, high protein diet, males > females, low fluid/dehydrated, recurrent UTIs, diabetes, gout, RTA’s,
hypercalcemia, some medications (allopurinol)
-kidney cancer is usually painless, discovered incidentally
-classic triad is flank pain, abdominal mass, and hematuria
-risk factors smoking, male, older, obesity, fam hx, exposure to heavy metals/chemicals
-bladder cancer usually is gross hematuria with urinary symptoms, same risk factors as kidney cancer but also chronic bladder irritation
-prostate cancer usually urinary symptoms like BPH, if metastatic disease can have bone pain, obstructive renal failure, weight loss
-risk factors are age >50, AA race, high fat diet, and fam hx

Pathophys
-glomerular hematuria implies blood is coming from kidney itself
-IgA nephropathy (Berger’s), Alport’s, thin glomerular basement membrane
-nonglomerular causes can be from kidney, ureter, bladder, or urethra
-most common locations of stones are ureteropelvic junction, where ureter crosses iliac vessels, and ureterovesical junction
-stones

-high protein diet lowers urinary pH from breakdown of protein, causing more excretion of uric acid and can make stones
-renal cancer
-most common is clear cell, also papillary and chromophobe
-most common location for mets are lungs
-mostly sporadic but can be from VHL (autosomal dominant, chr17), tuberous sclerosis (autosomal dominant, chr9), and Birt-
Hogg-Dube (autosomal dominant, chr7
-paraneoplastic syndromes
-polycythemia from increased EPO
-hypercalcemia from PTH-like hormone production
-HTN from increased renin
-Cushing’s from ectopic cortisol
-Stauffer’s can have reversible liver dysfunction, hepatosplenomegaly, elevated ALP, GGT, ESR
-bladder cancer most common type is urothelial cell carcinoma (transitional cell carcinoma)
-prostate cancer most common is prostatic adenocarcinoma

Workup
-for gross hematuria
-confirm hematuria  dipstick (pH, SG, protein, glucose, blood, ketones, nitrite, leukocyte esterase)
-microscopic UA for RBC/hpf, WBCs, bacteria presence, crystals
-if brown-colored urine, red cell casts, dysmorphic RBCs, proteinuria suggests glomerular source and should refer to
nephrology
-CBC, BUN/crt, PT/INR, PTT, PSA
-urine cx and cytology
-if nephrolithiasis, get CT w/o contrast
-if HIV, get CT with contrast since HIV drugs can cause small radiolucent stones that can be missed w/o contrast
-US for pregnant women, women of childbearing age
-if gross hematuria in age >50yo should do workup for malignancy
-CT urogram, urine cytology, cystourethroscopy

Management
-renal stones
-if <5mm will likely pass on their own, and can do supportive care, can do alpha blockers (tamsulosin) to relax ureteral wall

Page 71 of 77
-stones 5-9mm – managed by clinical judgment
-if >9mm require intervention
-if stones cause urosepsis, intractable pain, progressive renal damage, or patient only has 1 kidney give ureteral stent or
percutaneous nephrostomy tube placement

-renal masses (RCC)


-radical or partial nephrectomy
-if small masses can do ablation
-bladder cancer
-transurethral resection
-intravesicle chemo (mitomycin)
-high rate of recurrence, need to be closely monitored
-radical cystectomy if severe disease (males take out prostate and seminal vesicles too, females take out cervix, uterus, fallopian
tubes, and part of vagina)
-prostate cancer
-radical prostatectomy (removing prostate and seminal vesicles)
-nonsurgical is external beam radiation, brachytherapy (radioactive beads within prostate), anti-androgen therapy
-significant gross hematuria  put in foley to prevent urinary retention due to obstruction by blood clots
-irrigate bladder out with catheter tip syringe
-foley can tamponade bleeding if 2/2 to urethral/prostatic trauma
-never place foley in setting of trauma (pelvic fracture) when there is blood at urethral meatus  urethral injury, foley can completely
transect
-do cystourethrogram first

PART XVI: VASCULAR

Chapter 55: Transient loss of vision in right eye  embolus in ICA


Differential

H&P
-amaurosis fugax = transient vision loss, usually “curtain or shade” coming down, usually from atherosclerotic debris from plaque in
ICA that embolizes to ophthalmic artery (first branch of ICA in brain)
-can see Hollenhorst plaque  cholesterol microemboli seen in retinal arterioles that have bright, yellow, refractile appearance
-if transient motor/sensory loss of arm and leg, would be TIA if <24 hours, stroke if >24 hours, means ischemia to contralateral middle
cerebral artery, can also have aphasia if left sided MCA involved
-carotid bruit means turbulent flow, likely stenosis in ICA due to atherosclerosis
-other causes can be murmur transmitted from heart to neck, or stenosis of external carotids or subclavian
-can have syncope with bilateral carotid stenosis
-risk factors for carotid stenosis: older age, male, HTN, smoking, hypercholesterolemia, diabetes, obesity

Page 72 of 77
Anatomy
-ICA has no branches in neck, external carotid does
-external carotid branches: superior thyroid, ascending pharyngeal, lingual, facial, occipital, posterior auricular, maxillary, superficial
temporal
-carotid plaques go to anterior circulation and can go to MCA or ACA, if PCA then source should not be from carotids

Pathophys
-strokes are either ischemic or hemorrhagic
-ischemic is more common, from blockage to blood supply of brain, usually from ICA at carotid bifurcation and from LA
thrombus with a-fib, or thrombous from MI or endocarditis
-lacunar stroke is ischemic stroke of the deep penetrating arteries
-hemorrhagic from artery rupturing and bleeding, usually from poorly controlled HTN, trauma, AV malformations,
subarachnoid hemorrhage from aneurysm
-symptomatic carotid stenosis: ipsilateral amaurosis fugax and contralateral TIA/stroke symptoms

Workup
-carotid duplex scan
-CT angiogram or MR angiogram
-high grade stenosis mandates operation but once complete occlusion, no indication for carotid endarterectomy

Management
-in symptomatic patient:

-in asymptomatic patient

-can also do medical alone: aspirin, clopidogrel, and statin or endarterectomy or carotid artery stenting
-after stroke/TIA do carotid endarterectomy within 2 weeks to prevent further embolic events
-once artery fully occluded, no further flow in artery, no risk of embolization/stroke
-no role of endarterectomy if patient had a previous stroke with severe deficits since no further viable brain to protect and risk of surgery
outweigh benefits, goal of endarterectomy is to protect remaining functions
-symptomatic patients endarterectomy best when 70-99% occluded, asymptomatic patients with 80-99% stenosis
-women in general have more unfavorable outcomes from carotid endarterectomies
-peri-operative aspirin and statin, proper bp control before endarterectomy
-common nerves injured in carotid endarterectomy

Page 73 of 77
Chapter 56: Right calf pain with walking  peripheral arterial disease
Differential

H&P
-claudication = pain in leg with walking, relieved within few minutes of rest, reproducible with same walking distance
-risk factors for PAD: smoking, diabetes, HTN, hypercholesterolemia, age, male, obese, sedentary, fam hx, heart attack, stroke
-Rutherford classification of PAD: 0 asymptomatic, 1 mild, 2 moderate, 3 severe, 4 ischemic pain at rest, 5 minor tissue loss, 6 major
tissue loss
-ischemic rest pain when patient dangles foot or walks to relieve pain
-Buerger’s sign = advanced chronic ischemia, foot turns pale after elevated for 1-2min, then when danlged down after becomes ruborous
from marked arteriolar dilation, usually ABI <0.4
-ischemic rest pain ddx

-on exam can see pulse deficit, calf muscles atrophy, hair loss, loss of sweat glands, skin is shiny, ulcers can develop, CRT >2s
-superficial femoral artery (supplies calf) is most common site of plaque and common femoral artery most common site of local arterial
emboli

Pathophys
-dependent rubor is a sign of advanced chronic ischemia, arterioles no longer constrict in presence of increased hydrostatic pressure,
arterioles in foot become dilated to maximize blood delivery, blood pools in foot
-other causes of claudication are Buerger’s disease, chronic embolization, vasculitis, entrapment of popliteal artery
-claudication in upper extremities is usually subclavian steal syndrome where there is atherosclerotic plaque in subclavian artery causing
retrograde flow in vertebral artery during arm exercise, present with transiet vertigo, dizziness, syncope when arm exercising
-Buerger’s disease = thromboangiitis obliterans, in <40yo male smokers, where inflammatory process occludes distal arteries below
knee, in the hands, associated with amputation and lack of response to bypass, treatment is smoking cessation
-Leriche syndrome = slowly developing occlusion of infrarenal aorta usually in smokers
-triad: buttock/thigh claudication, absent femoral pulses, impotence

Workup
-ABI

Page 74 of 77
-normal 1-1.2
-mild disease 0.9-0.7
-moderate disease 0.7-0.4
-severe disease/rest pain <0.4 (usually ischemic rest pain)
-CT angiogram or MR angiogram if doing interventions
Prognosis
-if ischemic rest pain in foot, likely 50% will have limb loss in <1yr
-ischemic rest pain, nonhealing ulcer, gangrene is limb threatening, need more aggressive treatment

Management
-medical approach, smoking cessation and exercise, control HTN, DM, diet
-cilostazol (quinolinone derivative) that inhibits platelet aggregation and is a vasodilator, also inhibits thrombin formation, increases
HDL and lowers TG levels, contraindicated if HF patient
-statin, aspirin, clopidogrel
-can do endovascular angioplasty/stenting or open surgical endarterectomy or bypass
-Rutherford class 2 or 3 candidates for invasive treatments if medical management fails
-Rutherford class 4 and above go directly to invasive approach
-mortality is due to MI, stroke

Trouble
-forgetting to ask about ischemic rest pain (Rutherford class 4), since intervention is required
-ABI of 1.0 is not normal in diabetic patient; diabetic patients can have falsely elevated or falsely normal ABI since can have
Monckeberg’s arteriosclerosis causing rigid vessels from medial calcinosis, bp cuff unable to compress artery so pressure is falsely
elevated

Chapter 57: Sudden onset of severe left-sided abdominal pain  ruptured AAA
Differential

H&P
-risk factors: smoking, age, white, male, CAD, hx of aneurysms, atherosclerosis, fam hx, HTN, COPD
-diabetes actually protective against AAA
-screening with PE and abdominal US for men 65-75yo who have smoking hx
-first degree relative of patient with AAA, screened at age 60
-f/u annually if 3-4cm, f/u biannually if 4-4.5cm
-femoral and popliteal artery aneurysms also associated with AAA, usually in males

Pathophys
-AAA leads to degeneration of medial layer through degradation of elastin and collagen
-diameter of >3cm is considered an aneurysm
-ongoing smoking increases growth rate of AAA, DM patients have slower growth
-matrix metalloproteinases (MMPs) are for collagen turnover for inflammation/healing, patients with AAA have elevated MMP in aortic
wall which weakens the arterial wall and dilates aneurysm
-statins reduce MMP activity
-AAA rupture is usually on left
-risk factors for rupture: size, HTN, COPD, female gender
Page 75 of 77
-aortic dissection vs AAA

Workup
-ABCs, IV access
-permissive hypotension otherwise massive fluid resuscitation prior to OR exacerbates hemorrhage by dilating out factors
-if stable get CT angiogram
-if unstable  OR for ex lap

Management
-size
-3-4.4cm  annual US
-4.5-5.5cm  US q3months
->5.5cm  elective repair
-enlarging at >1cm/year or symptomatic  elective repair
-non-ruptured, asymptomatic AAA  repair is open vs endovascular
-main morbidity/mortality of either approach is MI
-ruptured AAA  open vs endovascular, open is better for unstable patient
-mortality is 80-90% for AAA rupture
-endovascular repair has risk of endoleak (persistent arterial flow into aneurysm sac, aortic aneurysm)
-need to monitor after endovascular approach with CT angio at 1 month and 1 yaer post-op then annually

Trouble
-assuming AAA is unruptured if no contrast extravasation on CT with adjacent retroperitoneal fluid next to AAA
-postop ischemic colitis from ligating inferior mesenteric artery can present with abdominal pain, leukocytosis, diarrhea, manage
supportively with NPO, IV fluids and abx
-not considering aortoenteric fistula for GI bleed after AAA repair

Chapter 58: Cold, painful right lower extremity  acute limb ischemia
Differential

Page 76 of 77
H&P
-6 P’s of limb ischemia: pain, pallor, paresthesia, pulselessness, paralysis, poikilothermia
-cardiac hx is important for source of emboli
-embolic causes are usually of cardiac origin (a-fib, or acute MI), thrombotic events usually in pre-existing atherosclerosis in leg arteries
-if long-standing ischemia can have ischemic ulcers, no hair, atrophic skin changes
-ask for hx of atherosclerotic disease, smoking, cholesterol levels, CAD disease, DM, claudication
-ask if previous interventions for PAD, because could be thrombosis of previous graft for PAD

Pathophys
-generally emboli lodge at arterial bifurcations (aortic, common femoral, popliteal)
-skeletal muscles show signs of irreversible cell damage after 3hours, complete cell damage at 6 hours

Workup
-Doppler (healthy peripheral artery is triphasic – systolic flow, early diastolic retrograde, forward diastolic), biphasic may be normal but
also early disease state, monophasic signals are abnormal and signify severe reduction in blood flow
-can do Duplex scanning
-CT angio for proximal to limb ateries
-TTE if cardiac suspicion, can do bubble study

Management
-heparin, IV fluids, dependent position
-Doppler or duplex US
-if normal motor/sensory (stage I) or only sensory deficit (stage IIa) can do endovascular therapy since may take 24-48hour to take
complete effect
-endovascular with catheter tPA and aspiration thrombectomy
-if motor deficit (stage IIb) open surgery for immediate revascularization
-if loss of motor/sensory, severe (stage III)  amputation
-if revascularization of ischemic tissue can cause reperfusion syndrome  toxic byproducts released from damaged tissue into
general circulation, rhabdo is a concern  renal failure, also hyperkalemia, can progress to ARdS, DIC, shock
-if cardioemboli should do surgical approach since can be located proximal to inguinal ligament
-also start long-term anticoagulation
-if thrombus, should prefer endovascular thrombolysis, since usually smaller and distal arteries

Trouble
-mistaking acute limb ischemia with neurological symptoms/diagnosis
-failure to monitor CPK and myoglobinuria
-if after revascularization patient develops peaked T waves on EKG  hyperkalemia, give IV calcium gluconate, bicarbonate, insulin
and glucose, albuterol, loop diuretics, kayexalate, dialysis

Page 77 of 77

Você também pode gostar